Reading Comprehension Practice

background image

Reading comprehension practice

This file contains 30 reading comprehension passages with

answered and explained questions.

Good luck on your test.

background image

!

"

#

$ %

&

!

'#

& &

(

)

&

*! !

+

,

&

*

*

-!

)*)

*&

'#

)

&

*! !

$

$

)

&

)

&

'

%

.

&

/

&

&

&

)

&

0

&

)

'#

&

) *&

'

1

&

%

0

)

'-

&

2

) 0

'.

&

-

&

&& &

&

1

&

)

&

&

&

)

)

'-

&

$

(

)

3

)

'.

)

&

-

&

$

&

'#

)

-

&

&

&

&

'

-

&

0

&

'

-

&

&

)

)

)

'-

&

)

)

&

)

)

&

$ )

'

#

)

-

&

$

)

)

(

'4

)

&)

0

0

& &

&

&

'

background image

4

5

(

)

&

*! !

)

' 4

$

)

0

&

&

(

&

'

'#

&

+., &&

+6,

&

&

*! !

+!,

&

*! !

+7,

&

+ ,

2

)

&

)

1

#

)

!'#

& ) &

)

&

)

&

*! !

)

&

&

'7

)

& '

8'.

&

%

.

&

+.,

& &

'

+6,

2

&

)

&

)

) &

+!,

)

&

*! !

+7,

)

&

*! !

+ ,

)

0

&

)

'

#

)

'.

&

%

.

& &

&

)

'

) '

#

)

'

9'4

5

-

&

&

:! ;#

+.,

(

&

+6,

+!,

+7,

+ ,

&

background image

#

)

.'#

3

0

1 #

4

&

'.

(

&

)

4

'

<'#

=

)

2

&

>

+.,

)

)

)

&

) 0

)

+6,

)

)

)

&

) 0

)

+!,

)

&

)

0

&

&

'

+7,#

&

*! !

&

*

*

)*)

*&

'

+ ,

&

*! !

'

#

)

'#

&

&

*! !

'

)

)

'

background image

Take a very commonplace, often discussed and critical topic: Are we detecting a

greenhouse effect, and related to this, is it exacerbated by "homogenic factors," i.e.,

human actions? Most would be inclined to give a positive answer to both of these

questions. But, if pushed, what would be the evidence, and how well grounded would

it be for such affirmations?

Within scientific communities and associated scientifically informed circles, the

answers have to be somewhat more ambiguous, particularly when rigorous questions

concerning evidence are raised. Were scientific truth to be a matter of consensus, and

some argue that scientific truth often turns out to be just that, then it is clear that there

is beginning to be a kind of majority consensus among many earth science

practitioners that the temperature of the Earth, particularly of the oceans, is indeed

rising and that this is a crucial indicator for a possible greenhouse effect.

Most of these scientists admit that the mean oceanic temperature has risen globally in

the last several decades. But this generalization depends upon how accurate

measurements may be, not just for samples, but also for the whole Earth. Hot spots,

for example the now four year old hot spot near New Guinea which is part of the El

Niño cycle, does not count by itself because it might be balanced by cold spots

elsewhere. And the fact of the matter is that "whole earth measurements" are still rare

and primitive in the simple sense that we simply do not have enough thermometers

out. Secondly, even if we had enough thermometers, a simply synchronic whole earth

measurement over three decades is but a blip in the diachronic history of ice age

cycles over the last tens of thousands of years. Thirdly, even if we know that the

earth is now heating up, has an ever increasing ozone hole, and from this strange

weather effects can be predicted, how much of this is due to homogenic factors, such

as CFCs, CO2 increases, hydrocarbon burning, and the like? Is it really the case, as

Science magazine claimed in l990, "24% of greenhouse encouraging gases are of

homogenic origin"?

1. In this passage the author is primarily interested in

(A) whether scientific truths are simply a matter of consensus

(B) determining how well established the greenhouse effect is and to what degree it is

worsened by human actions

(C) whether the hot spot El Niño is balanced elsewhere by cold spots

(D) determining if most scientists would be inclined to give a positive answer to the

question of whether there is a greenhouse effect and if it is worsened by human

actions

(E) making a simple synchronic whole earth measurement more than a blip in the

diachronic history of Ice Age cycles over the last tens of thousands of years.

The best answer is B. The author questions the claim that there is indeed a

greenhouse effect that is made worse by human actions. (A) is too general an answer,

while (D) is too specific. (D) is wrong because it is probing whether scientists agree,

not whether there the phenomenon actually exists.

background image

2. The author of the passage would be most likely to agree with which of the

following statements about the greenhouse effect?

(A) 24% of greenhouse encouraging gases are of homogenic origin.

(B) there is a greenhouse effect that is exacerbated by homogenic factors.

(C) The ozone hole is increasing due to homogenic factors, such as CFCs, CO2

increases, hydrocarbon burning, and the like.

(D) One can determine if mean oceanic temperatures have risen globally in the last

several decades only if measurements of ocean temperatures are precise.

(E) Hot spots, such as the El Niño cycle, should not be counted as a factor in the

greenhouse effect.

The best answer is D. Scientists are basing their claims on global warning on rising

ocean temperatures. One can tell if temperatures have in fact risen only by measuring

them correctly.

3. It can be inferred from the passage that

(A) we cannot be certain that strange weather effects are a result of the earth heating

up and an ever increasing ozone hole

(B) the greenhouse effect is the most widely discussed topic in the scientifically

informed circles

(C) If the temperature of the oceans has ceased to rise at an ever increasing rate, then

the rate of global warming has increased

(D) strange weather effects have been shown to be due to the diachronic effects of

hydrocarbon burning and not to increases in CFC.

(E) Strange weather effects are caused by the increase use of CFCs, CO2, and similar

gasses.

The best answer is A. The author is questioning the cause and effect relationship

between the increasingly large ozone hole and global warming, as well as cause and

effect relationship between global warming and strange weather effects.

4. The author’s claim that, a simply synchronic whole earth measurement over three

decades is but a blip in the diachronic history of ice age cycles over the last tens of

thousands of years would be strengthened if the author

(A) indicated the minimum number of thermometers necessary for a whole earth

measurement

(B) described the factors that precipitated the start of a new ice age

(C) compare synchronic whole earth measurements with diachronic whole earth

measurements

(D) proved that the mean number of years required to detect significant changes in

weather patterns is greater than thirty

background image

(E) specified the exact location and quantity of thermometers placed by scientists

around the globe

The best answer is D. If one knows that change can be detected only after much more

than thirty years, then measurements taken over a thirty year period is insignificant

background image

. .

&

.

!

%

0 &

+.!% ,

))

0

&

&

0

&

'#

0 &

&

)

?@

0 & &

5

&

&

0 &

2

&

0 '#

.!%

)

&

0

%

0 &

2

&

0

'

% )

&

3

(

0

3

) '#

&

0

0

&

&

)

)

0

0

'#

0

&

&

)

&

&

'

#

.!%

)

&

A

B

&

$

(

&

)

&

'.

&

0 &

&

0

0

8?*9?

&

&

&

0

0

8?*9?

&

&

&

'

)

.!%

&

&

&

0

&

'

&

.!%

)

)

)

'

)

3

)

&

(

&

0

'

background image

C

0

&

'%

0

0

)

&

&

&

)

&

&

'C

&

&

&

0

(

)

&

'

'

)

&

+.,#

&

&

&

&

&

0 & &'

+6,#

0 &

&

'

+!,#

0 & &

&

&

)

'

+7,#

&

)

0

0

'

+ ,#

0

0

&

0 &

0

&

'

#

)

!'4

)

+.,+6,

+7,

& '+ ,

)

&

0

0

&

0 &

'

8'

#

A

B

+.,.

&

&

)

$

(

'

+6,.

&

&

)

$

(

'

+!,

$

(

&*

&

)

'

background image

+7,#

.!%

&

&

)

&

)

3

)

&*

0 &'

+ ,#

.!%

&

&

)

&

)

3

)

&*

0 &'

#

)

'#

)

.!%

) 0

)

&

&

) ( '

9'

4

&

)

&

)

.!% >

+.,#

(

&

'

+6,#

&

(

'

+!,.

9???

&

)

&

& '

+7,.

9???

&

)

0 &

& '

+ ,.

@

0

0

&

0 &

0 &

&

'

#

)

6'

0

&

0

)

)

(

)

0

)

)

.!%

&'

<'

#

&

1 #

& )

0 &

& '

background image

+.,!

&

&

'

+6,#

&

&

)

0

3

) '

+!,!

&

3

'

+7,!

&

'

+ ,!

&

(

&

'

#

)

.'#

&

&

&

&

&

'

background image

Gene therapy offers a new treatment paradigm for curing human disease. Rather than

altering the disease phenotype by using agents that interact with gene products, or are

themselves gene products, gene therapy can theoretically modify specific genes

resulting in disease cure following a single administration. Initially gene therapy was

envisioned for the treatment of genetic disorders, but is currently being studied for use

with a wide range of diseases, including cancer, peripheral vascular disease, arthritis,

Neurodegenerative disorders and other acquired diseases.

Certain key elements are required for a successful gene therapy strategy. The most

elementary of these is that the relevant gene be identified and cloned. Upon

completion of the Human Genome Project, gene availability will be unlimited. Once

identified and cloned, the next consideration must be expression of the gene.

Questions pertaining to the efficiency of gene transfer and gene expression remain at

the forefront of gene therapy research, with current debates revolving around the

transfer of desired genes to appropriate cells, and then to obtaining sufficient levels of

expression for disease treatment. With luck, future research on gene transfer and

tissue-specific gene expression will resolve these issues for the majority of gene

therapy protocols.

Other important considerations for a gene therapy strategy include a sufficient

understanding of the pathogenesis of the targeted disorder, potential side effects of the

gene therapy treatment, and a more in depth understanding of the target cells which

are to receive gene therapy.

Gene transfer vector is the mechanism by which the gene is transferred into a cell.

Currently there are at least 150 clinical gene therapy protocols worldwide. Since the

approval process for these protocols is not as public outside the U.S., it is difficult to

ascertain the exact number of worldwide protocols. As of December 1995, 1024

patients had been treated with either a gene transfer or gene therapy protocol. Much

controversy exists regarding how many of these patients have benefited from their

gene therapy, and no one has yet been cured.

Public controversy in the field of human gene therapy is driven by several factors.

Ordinary citizens as well as scientists easily understand the enormous potential of

gene therapy, but the former may not appreciate all the pitfalls and uncertainly that lie

in the immediate future. The financial interests of biotechnology firms and, some have

asserted, the career interests of some gene therapists have encouraged extravagant, or

at least overly optimistic public statements about contemporary gene therapy. In spite

of the proliferation of protocols, the actual number of patients treated remains small,

and only one genuinely controlled study of human gene therapy has been published as

of this date.

1. In the passage, the author anticipates which of the following as a possible obstacle

to the introduction of gene therapy to mainstream medicine?

(A) Overly optimistic public statements given by scientists who have a vested interest

(B) The general public’s difficulty in grasping gene therapy’s vast potential.

(C) Unchecked financial interests of biotechnology firms.

background image

(D) The relatively small number of controlled studies of human gene therapy

published as of this date.

(E) Hazards of which the general public is currently unaware.

The best answer is E. The passage states that both ordinary citizens and scientists

understand potential of gene therapy, but the former, i.e. the general public may not

appreciate all the pitfalls and uncertainty that lie in the immediate future.

2. All of the following are mentioned in the passage as elements that are required for a

successful gene therapy strategy EXCEPT:

(A) Identifying the relevant gene

(B) Expressing the relevant gene

(C) Determining the side effects of the relevant gene

(D) Understanding of the pathogenesis of the targeted disorder

(E) Gaining and a more in depth understanding of the target cells which are to receive

gene therapy.

The best answer is C. One must determine the side effects of the relevant gene therapy

treatment, and not of the relevant gene itself.

3. The author’s attitude toward the gene therapy as a future cure for cancer,

peripheral vascular disease, arthritis, Neurodegenerative disorders and other acquired

diseases is

(A) Indifference

(B) Disapproval

(C) Amusement

(D) Cautious optimism

(E) Censure

The best answer is D. In the opening sentence, the author optimistically states that

gene therapy offers a new treatment paradigm for curing human disease. However, in

the fifth paragraph the author mentions some of the problems.

4. The primary function of the fifth paragraph is to

(A) Explain effects

(B) Recommend actions

(C) Identify problems

(D) Evaluate solutions

(E) Warn of consequences

The best answer is C. The fifth paragraph is devoted to identifying various problems

in the field today, such as the exaggerated claims made by biotechnological firms with

vested financial interests.

background image

Unlike the United States with its generalissimo politicians - Washington, Jackson,

Grant, and Eisenhower- the ‘martial arts’ have been conspicuously absent from

Canadian politics. The exception to the rule is former Prime Minister Pierre Elliot

Trudeau, who became the first Canadian leader to bring a gunslinger ethos to

Canadian politics. Trudeau introduced Canada to the refined art of single combat; it

was the politics of “doing it my way”. Single-combat confrontation implied much

more than the renegade in power did, and far less than the tricks of William Lyon

Mackenzie King, prime minister intermittently between 1921 and 1948.

Trudeau’s unique background prepared him for the role of authoritarian leader he

would assume later in life. Born on October 18, 1919, Trudeau lived in French-

speaking Montreal, but heard English at home from his mother, making it easy for the

young politician to appeal to all sectors of Canada, a bilingual country. As a young

man, he walked and cycled through Europe, finding himself on occasion on the wrong

side of the bars in foreign jails. By 1940, Trudeau entered the law faculty at the

University of Montreal. As a student he enlisted in the Canadian Officers Training

Corps, where he was given a commission as a lieutenant, a rank he held until his

retirement in 1947. Trudeau, a renowned sportsman, held a brown belt in karate,

knew how to skin dive and could descend 150 feet off a cliff with ease. He continued

performing flamboyant physical feats even in later life as Canada’s fifteenth prime

minister, astounding Canadians with his prowess. The public’s adoration made it

possible for him to practice his personal brand of ‘do it my way’ politics, initiating

profound and long-lasting changes to his country.

Other leaders would never have undertaken to deal with such taboo issues as divorce,

abortion and homosexuality– matters likely to infuriate conservative Canada from

coast to coast. Even the powerful Mackenzie King dared not touch any of the three,

though Trudeau tackled them together in an omnibus bill as Minister of Justice under

Lester B. Pearson. His reason for loosening legislation on these issues was, as he put

poetically put it, "The state has no business in the bedrooms of the nation." The

myths-makers have it that this was Trudeau's first deliberate ‘gun slinging’ move,

performed with the ultimate goal of attaining national leadership. Contrary to popular

belief, Trudeau had no leadership aspirations at the time; all he had was a passion for

combat that eclipsed other religious considerations. Trudeau instigated far-reaching

changes in legislation governing divorce, abortion and homosexuality that have had a

major impact on Canada, shaping the country into what it is today.

1. The primary focus of the passage is on which of the following?

(A) Comparing two Canadian prime ministers and contrasting their personal style of

leadership

(B) Describing the leadership style of one of Canada’s prime ministers

(C) Evaluating the success of the leadership style of one of Canada’s prime ministers

(D) Summarizing the contribution of one of Canada’s prime ministers

(E) Tracing the long-term impact of legislation put forward by one of Canada’s prime

ministers

The best answer is B. The passage describes the ‘doing it my way’ type leadership

style of former Prime Minister Pierre Elliot Trudeau.

background image

2. It can be inferred from the passage that former Prime Minister William Lyon

Mackenzie King

(A) was opposed to abortion and put forward legislation making it illegal

(B) was opposed to abortion but did not put forward legislation making it illegal

(C) was in favor of abortion and put forward legislation making it legal

(D) was in favor of abortion but did not put forward legislation making it legal

(E) did not put forward legislation making abortion legal

The best answer is E. The passage states that Mackenzie King dared not touch the

issue of abortion, so it can be inferred from the passage that he did not put forward

any abortion legislation. The passage provides no information on what Mackenzie

King’s opinions were on the subject.

3. The author of the passage mentions Trudeau’s accomplishments in sports primarily

in order to

(A) Explain the source of Trudeau’s physical stamina

(B) Illustrate that he had earned the adoration of the Canadian public

(C) Contrast it to his personal brand of ‘do it my way’ politics

(D) Provide one reason why he was able to single-handedly push through legislation

that should normally have shocked conservative Canada

(E) Provide one reason why he was able to single-handedly thwart legislation that

should normally have shocked conservative Canada

The best answer is D. One of the ways Trudeau won the hearts and minds of the

Canadian public was by astounding them with his physical prowess. Because the

public was so awed by Trudeau as an individual, he was able to pass legislation

considered progressive.

4. The quotation "The state has no business in the bedrooms of the nation," is most

probably used to

(A) present the opinion that the state should have less of a say in issues that are

essentially not public matters

(B) present the opinion that the state should have less of a say in issues that are

essentially public matters

(C) present the opinion that the state should have more of a say in issues that are

essentially not personal matters

(D) provide an example of Pierre Elliot Trudeau’s flamboyant style that he used to

captivate the Canadian public

(E) contrast Pierre Elliot Trudeau’s flamboyant style with that of Lester B. Pearson

The best answer is A. Matters that concern “the bedrooms of the nation” are private

matters. Trudeau sought to reduce government control in such matters. Answers (B)

and (C) say the opposite, namely that government control in public matters

background image

Art is visible. However, everything one sees is filtered through certain conditions,

some of them historical, and others, natural. The historical conditions include the

material which is used — oil, colors, and the canvas; second, a certain style, i.e., a

system of rules by which things visible are submitted a priori. There can be a general

style, for example, the style of Impressionism, or a particular style, for example, the

individual ways in which two painters, both impressionists, paint. The natural

conditions include certain unchanging psychological laws of sight, for instance, the

effects of colors or optical illusions.

The conditions of art are nothing but a particular way of interpreting reality. To

understand this, one can examine the difference between the classical Greek and the

classical Egyptian styles. For the Greeks, the reality of the visible was given by the

perspective and the situation in which the object appears; for that reason they

presented a person in his individual movements. For the Egyptians, however, this was

only the appearance of a transitory moment, which, according to their beliefs, was not

real. Therefore, the Egyptians searched for the permanent essence and the typical

character in their depiction of an object. For the Egyptians, Greek art was an illusion;

for the Greeks, on the other hand, Egyptian art was unrealistic constructivism.

The way in which reality appears in art must not be regarded on its own. It is affected

by many other systems of recognizing reality, including the political, religious,

economic, intellectual, and social — in short, all the phenomena of human life.

Moreover, art is always of a certain epoch, with its particular conception of reality.

Thus, when discussing, for example, the art of ancient myth, of medieval Christianity,

or that of the technological age, one must be aware that myth, Christianity, or

technology was the most salient feature of the epoch.

It is paradoxical to understand art as some kind of copy of the fields of experience

connected with it. So, for example, it is meaningless for the work of art as such if one

compares the landscape of a painting with the landscape, which served the artist as his

model. Even if the artist had tried to make what he painted as similar as possible to

the model he used, the landscape which he saw is only the matter from which

something completely different emerges since he has submitted its view to the a priori

conditions of art: namely to the material used (colors, canvas, etc.), to his style, and

even to the fact that he paints on a flat surface. Thus one must contemplate a work of

art by itself. Even if it is connected to other fields of experience it nevertheless

displays something unique which appears in that piece of art and there alone.

1. According to the passage, classical Egyptians did not present a person in his

individual movements

(A) Because the Greeks believed that the reality of the visible was given by the

perspective and the situation in which the object appears

(B) Because the Greeks did not believe that the reality of the visible was given by the

perspective and the situation in which the object appears

(C) Because the Egyptians believed that the reality of the visible was given by the

perspective and the situation in which the object appears

(D) Because the Egyptians did not believe that the reality of the visible was given by

the perspective and the situation in which the object appears

(E) Because for the Egyptians, Greek art was an illusion.

background image

The best answer is D. The Egyptians believed that the reality of the visible was not

given by the perspective and the situation in which the object appears. They thought

that this was the appearance of a transitory moment, which was not real.

2. The author mentions which of the following as one of the conditions through

which art is seen?

(A) Impressionism

(B) Optical illusions

(C) Nature

(D) Perspective

(E) Illusions

The best answer is B. The author states in the first paragraph that art is seen through

natural conditions including optical illusions.

3. By asserting that art is filtered through certain conditions (line???), the author

suggests which of the following?

(A) Even if the artist tried to make what he painted as similar as possible to the model

he used, he would not succeed

(B) Even if the artist tried to make what he painted as similar as possible to the model

he used, it would be impossible to critique it

(C) Even if the artist tried to make what he painted as similar as possible to the model

he used, the landscape would be affected by many other systems of recognizing

reality

(D) The way in which reality appears in art influences other systems of recognizing

reality, including the political, religious, economic, intellectual, and social

(E) The way in which reality appears in art is influenced by other systems of

recognizing reality, including the political, religious, economic, intellectual, and

social

The best answer is E. The author states in paragraph three that the way in which

reality appears in art is affected by many other systems of recognizing reality,

including the political, religious, economic, intellectual, and social — in short, all the

phenomena of human life.

4. The author cites the example of psychological laws of sight, for instance, the

effects of colors or optical illusions in order to illustrate

(A) Laws of nature, which are not affected by history

(B) Laws of nature, which are affected by history

(C) Laws of nature, which change history

(D) Laws of history, which are affected by nature

(E) Laws of history, which are not affected by nature

The best answer is A. The author details two kinds of conditions, historical and

natural. The natural conditions are said to be unchanging, therefore not affected by

history.

background image

For a generation of suppressed, restless, working-class youths living in 1960 Jamaica,

ska was a medium through which they could find expression. Since its original

appearance, ska has resurfaced twice, each time presenting itself in a different guise to

a new generation of music aficionados. Overcoming its humble beginnings, it has

become one of the twentieth century’s most enduring and influential styles of music.

Since the early 1940's, Jamaica had adopted and adapted many forms of American

musical styles. The predominantly black inhabitants of Jamaica took a liking to

rhythm and blues music, importing a considerable number of American records that

were showcased at dance halls in the early 1960s. Jamaican musicians took up the

elements of rhythm and blues and combined it with traditional Jamaican mento music.

The result was the first wave of ska.

Musically, ska is a shuffle rhythm similar to mento but with even closer ties to rhythm

and blues, placing the accent on the second and fourth beats, often moving in a 12-bar

blues frame. The after beat, played on the piano or strummed by a rhythm guitar,

came to be characteristic of the form. A horn section, usually consisting of trumpets,

trombones, and saxophones, was a vital element. Classic bands, such as the Wailers

wrote songs written about Trench Town (a ghetto), rude boys (street thugs), romance,

and even religious themes. In 1965, ska began to take a backseat to a newly evolved

type of music, called rock steady, which was more dependent than ska had been on

rhythm provided by the bass guitar and drums.

Ska was later exported by traveling Jamaican artists to Great Britain, where it became

known as "blue beat." By the mid 1970's, early British punk bands were infusing

reggae, a style of music that came from rock steady, into their music. Near the end of

the decade, however, there was a resurgence of the influence of ska because of its

upbeat, danceable rhythm. This faster paced ska came to be known as two tone. One

of the essential messages of two-tone ska was the promotion of racial harmony and of

having fun in the face of subjugation.

The third wave of ska began in America around 1990. Bands influenced by the two-

tone ska scene began to use punk and metal music to a greater extent. The

combination, which is much faster than two tone, sounds very different from the

original Jamaican brand of ska.

In its three different waves, ska has given voice to seemingly voiceless, downtrodden

generations. Each time it resurfaces, a new message is taken up, however, the old

messages are never forgotten.

1. The primary purpose of this passage is to

Contrast the musical rhythm of two-tone music with original Jamaican ska from

which it developed

Illustrate various ways in which rhythm and blues has influenced ska music

Outline the influences on the various forms of ska music from its inception in Jamaica

in early 1960s through its third wave in the 1990s

Describe events leading to the inception, rise and final demise of Jamaican ska music

Trace to evolution of ska music from its inception in Jamaica in early 1960s through

its third wave in the 1990s

background image

The best answer is E. The passage follows the development of ska. (D) is incorrect

because the passage does not deal with the demise, or death, of ska music.

2. According to the passage, Ska music has

Been influenced by rhythm and blues, mento and blue beat

Been influenced by rhythm and blues, and has influenced mento and rock steady

Been influenced by rhythm and blues and rock steady and has influenced punk and

metal musicians

Influenced rhythm and blues, reggae and metal musicians.

Been influenced by mento music and has influenced punk and metal musicians.

The best answer is E. Ska music has been influenced, among other things, by mento

music. In paragraph five, it is mentioned that ska musicians have influenced both

punk and metal musicians.

3. Which of the following statements about ska music is supported by information in

the passage?

Rock steady is more dependent than ska on the rhythm provided by the bass guitar

and drums.

Reggae, which counts ska as one of its primary influences, developed only after it was

exported by traveling Jamaican artists to Great Britain

Ska’s appeal over the last half century has been limited to voiceless, downtrodden

generations.

Two-tone is a faster paced form of ska that developed in the late 1970s

Mento music places the accent on the second and fourth beats, often moving in a 12-

bar blues-frame.

The best answer is D. In paragraph four, it says that this faster paced ska came to be

known as two tone.

4. The passage suggests that two tone music

I. Resurged near the end of the 1970s

II. Influenced bands in America in the 1990s

III. Promoted of racial harmony and of having fun in the face of oppression.

I only

II only

III only

II and III only

I, and II, and III

The best answer is E. (E) is the best choice because all of the facts presented above

are supported by the passage.

background image

Coca-Cola, which sold 10 billion cases of soft drinks in 1992, now finds itself asking,

where will sales of the next 10 billion cases come from? The answer lies overseas,

where income levels and appetites for Western products are at an all time high.

Often, the company that gets into a foreign market earliest dominates that country's

market. Coke patriarch Robert Woodruff realized this and unleashed a brilliant ploy to

make Coke the early bird in many of the major foreign markets. At the height of

World War II, Woodruff proclaimed, “Wherever American boys were fighting, they'd

be able to get a Coke.” By the time Pepsi tried to make its first international pitch in

the 1950s, Coke had established its brand name along with a powerful distribution

network.

During the last 40 years, many new markets have emerged. In order to tap into these

opportunities, both coke and Pepsi have attempted to find ways to cut through the red

tape that thwarts their efforts to conduct business in these new regions.

One key maneuver in the soda wars occurred in 1972, when Pepsi signed an

agreement with the Soviet Union that made it the first Western product to be sold to

consumers in Russia. This landmark agreement gave Pepsi the upper hand. At present,

Pepsi has 23 plants in the former Soviet Union and is the leader in the soft-drink

industry in Russia. It outsells Coca-Cola by a ratio of 6 to 1 and is seen there as a

local brand, similar to Coke’s homegrown reputation in Japan. However, Pepsi has

also encountered some obstacles. An expected increase in brand loyalty for Pepsi

subsequent to its advertising blitz in Russia has not materialized, even though Pepsi

produced commercials tailored to the Russian market and sponsored televised

concerts.

Some analysts believe that Pepsi’s domination of the Russian market has more to do

with pricing. While Pepsi sells for 250 Rubles (about 25 cents) a bottle, Coca-Cola

sells for 450 Rubles. Likewise, Pepsi sells their 2 liter economy bottle for 1,300

Rubles, while Coca-Cola’s 1.5 liters is marketed at 1,800 rubles. On the other hand,

Coca-Cola only made its first inroads into Russia 2 years ago. What's more, although

Coca-Cola's bottle and label give it a high-class image, Russians do not perceive

Coca-Cola as a premium brand in the Russian market. Consequently, it has so far

been unable to capture a market share.

1. According to the passage, all of the following have been used to attract customers

to buy a one of the two brands of soft drink mentioned in the passage EXCEPT

Offering soft drinks for a limited time at specially reduced prices

Sponsoring televised concerts

Designing a bottle and label to create a high-class image

Staging an advertising blitz including commercials tailored to the local market

Being the first country to enter a foreign market

The best answer is A. Answers (b), (C), (D) and (E) are all mentioned in the passage

as ploys used by either Pepsi or Coca Cola to attract new customers

2. The passage suggests which of the following about the Russian soft drink market?

background image

Price is an unimportant factor in the Russian soft drink market

Two liter economy bottles are more marketable than 1.5-liter economy bottles,

especially those sporting a high-class image.

One and a half liter economy bottles are more marketable than two liter economy

bottles, if sold at a lower price.

Russian consumers are more likely to purchase a product if the perceive it to be a

local brand

The Russian soft drink market is saturated with local brands.

The best answer is D. The passage states that one of the factors contributing to

Pepsi’s success in Russia is its perception by the public as a local brand.

3. The primary purpose of the passage is to

Review the marketing history of two soft drink giants

Contrast two different approaches to marketing soft drinks in the global market

Refute the traditional explanation for Pepsi’s success in the Russian soft drink market

Compare how well two soft drink companies have succeeded in a new foreign market

Explain why two soft drink companies have succeeded in a new foreign market

The best answer is D. The passage mainly compares Pepsi’s success in a new foreign

market, Russia, with Coca Cola’s relative failure. (A) is too general. (B) is incorrect

because both companies have the same general approach.

4. Which of the following best describes the relation of the first paragraph to the

passage as a whole?

It poses a question to be answered

It outlines an objective whose attainment will be discussed

It outlines a process to be analyzed

It advances and argument to be disputed

It introduces conflicting arguments to be reconciled

The best answer is B. The first paragraph asks and answers the question of where the

sale of the next 10 billion cases will come from, namely foreign markets. The rest of

the passage discusses ways in which the objective of conquering foreign markets is

accomplished.

background image

With the proliferation of electronic technologies in the latter part of the twentieth

century, many aspects of cultural practice have been redefined. The eradication of

physical boundaries that limit discourse and information access has had profound

effects upon the manner in which we conduct democracy. Yet, opinions strongly

differ over whether or not the growth of electronic networks will result in expanded

democracy. On one side of the debate are anti-utopians who fear that with the

intrusion of the Internet into many facets of life, personal freedom will be impeded

and the existing rift between the "haves" and "have-nots" in society will grow. On the

other side, many 'cyber-utopians' believe that new technologies can eliminate the

democracy of elected representatives with which so many people are dissatisfied. The

Internet, they say, will allow for a true participatory democracy in which citizens can

govern themselves without the interference of bureaucrats and legislators.

Neither of these theories by themselves can fully address the role of democracy in the

age of information. As debates about censorship and encryption have shown,

government regulation of the Internet can result in violations of the basic rights of

speech set forth in the constitution of the United States. Yet, groups that preach ‘Big

Brother’ theories of paranoia tend to neglect the fact that new technologies can help

balance the injustices of traditional power found in a centralized government. At the

same time, the likelihood of doing away with the present system of democracy in

favor of complete and pure self-governance seems impossible, and likely undesirable.

Both arguments about the future of the way in which discourse will occur highlight

the inherent relationship between communication and democracy. Perhaps a more

useful model for the study of this dynamic can be found in the model of the public

sphere proposed by Jorgen Habermas. In this realm, free and diverse equals come

together to deliberate and discuss pertinent issues without the impediment of external

coercion. The ensuing dialogue transpires in a profoundly democratic forum. The

dispensing of traditional hierarchies that occurs on the Internet appears to make

possible the type of categories necessary for Habermas ‘ideal speech situation to

occur.

However, postmodern critics indicate that the autonomous individual no longer exists

in a world where our identities are constructed as much for us as by us. And indeed,

much of the postmodern notion of self seems to fit closely with reconfigurations of

the subject brought on by electronic technologies. The question that arises then is,

how might the reconfiguration of communication enabled by the Internet work to

create a new form of 'cyber-democracy’ that better represents citizens' interests?

1. According to the passage, the 'cyber-utopians' mentioned in the passage would

most likely be in favor of which of the following innovations?

Every new legislation would be voted by every registered voter on the Internet

Government would increase the regulation of the Internet to include a curtailing of

politically biased messages

Government would decrease the regulation of the Internet including regulation of

politically biased messages

Discourse in legislative assemblies would be broadcast over the Internet

New technologies would gradually replace all forms of democracy

The best answer is A. According to the passage, 'cyber-utopians' believe that through

using the Internet, there could be a true participatory democracy, meaning that all

citizens, not just elected representatives could make legislative decisions.

background image

2. The passage supports which of the following statements about government

regulation of the Internet?

Government regulation of the Internet can result in infringements upon citizen’s

constitutional rights of free speech

Government regulation of the Internet can ensure against infringements upon citizen’s

constitutional rights of free speech

Government regulation of the Internet will make pure self-governance possible

Government regulation of the Internet will promote new technologies that can help

balance the injustices of traditional power

Government regulation of the Internet will eradicate physical boundaries that limit

discourse and information

The best answer is A. As stated in paragraph three, government regulation of the

Internet can result in violations of the basic rights of speech set forth in the

constitution.

3. The author is primarily concerned with

Advocating the use of the electronic technologies to improve democracy

Challenging the assumptions on which a theory of modern democracy is based

Describing events leading to the discovery of democratic uses of electronic

technologies

Explaining the importance of electronic technologies to modern politics

Examining the relationship between Internet communication and democracy

The best answer is E. The answer is not (A) because the author does not reach any

conclusions. (D) is incorrect because it does not discuss modern politics in general.

4. According to the passage, which of the following is considered by postmodern

critics to be a threat to the notion of self?

The interference of bureaucrats and legislators.

The proliferation of electronic technologies.

Reconfigurations of the subject brought on by electronic technologies.

Traditional hierarchies that occur on the Internet.

The impediment of external coercion.

The best answer is C. In the last paragraph, it says that much of the postmodern notion

of self seems to fit closely with reconfigurations of the subject brought on by

electronic technologies.

background image

Men are primarily and secondarily socialized into believing certain characteristics are

definitive in determining their masculinity. These characteristics range from playing

violently to not crying when they are injured. The socialization of masculinity in our

society begins as early as the first stages of infancy, with awareness of adult gender

role differences being internalized by children as young as two years old.

Studies show that advertising imagery equates masculinity with violence by

portraying the trait of aggression as instrumental to establishing their masculinity.

Lee Bowker, who researched the influence of advertisements on youth, asserts that

toy advertisements featuring only boys depict aggressive behavior and that the

aggressive behavior produces positive consequences more often than negative.

Bowker also looked at commercials with boys that contain references to domination.

His results indicated that 68.6% of the commercials positioned toward boys contain

incidents of verbal and physical aggression. However there were no cross gender

displays of aggressive behavior. Interestingly, not one single-sex commercial

featuring girls showed any act of aggression. Bowker’s research helps explain that it

is not just the reinforcement of a child’s close caretakers that lends legitimacy to

aggressive masculine tendencies but society as a whole, using the medium of

television.

William Pollack, a Harvard clinical psychologist, talks about how males have been

put in a "gender straightjacket" that leads to anger, despair and often violence. Pollack

states that society asks men to put a whole range of feelings and emotions behind a

mask and shames them if they display any emotion. Pollack contends that boys are

‘shame phobics’, even killing, in extreme cases, to avoid dishonor. It appears that the

standard defined by society allows men to express their emotion only through anger.

Ironically, though these rigid stereotypes of what it means to be a man have been

inculcated from an early age, men are often criticized for being one-dimensional in

their behavior and emotions.

Women often verbalize a desire for males to be sensitive and express their emotions.

But male insensitivity is the culmination of a societal indoctrination begun at birth.

Realistically, men are in a damned if they do, damned if they don’t situation. If they

fail to show their emotions, they are berated for being detached from the essence of

what constitutes a human being. On the other hand, if a male decides to expose his

emotions, he is often branded effeminate and regarded as inferior to other males who

stick closer to their gender’s traditional doctrine.

1. According to the passage, the television commercials examined by Bowker

Showed boys in more acts of verbal and physical aggression than of domination

Showed boys in more acts of domination than of verbal and physical aggression

Showed boys in acts of verbal and physical aggression only towards other boys

Showed boys in acts of verbal and physical aggression only towards other girls

Showed boys in acts of verbal and physical aggression towards other boys and girls

The best answer is C. Bowker’s research did not find any cross gender displays of

aggressive behavior, i.e. aggression of one gender to another

2. According to Pollack, one of the reasons for male violence is that

Society shames men who display feelings and emotions other than anger

Men kill in extreme cases to avoid dishonor

background image

Men are often criticized for being one-dimensional in their behavior and emotions

Society uses television as a symbol of its desires

Reinforcement from child’s close caretakers lends legitimacy to aggressive masculine

behavior

The best answer is A. (B) is incorrect because it does not give a reason for violence.

(C) is a result of the conditioning that leads to violence, not a reason. (D) and (E) are

incorrect because they are not opinions expressed by Pollack.

3. The passage suggests that, when compared with television advertisement featuring

boys, advertisements that had only girls were found

To have more references to domination

To be 68.6% less aggressive

To be remarkably similar in focus and content

To be replete with extensive examples of cross gender aggression

To be void of any acts of aggression

The best answer is E. Bowker found that not one single-sex commercial featuring

girls showed any act of aggression.

4. Pollack uses the term ‘gender straightjacket’ to emphasize

The narrow range of emotion that society allows men to express

The broad range of emotion that society allows men to express

The danger of anger, despair and violence towards men

The danger of anger, despair and violence perpetrated by men

The wide range of feelings that men actually experience

The best answer is A. Society does not allow men to act in ways it has deemed

inappropriate, hence Pollack considers the male gender to be in a ‘straightjacket’.

background image

Juror anonymity was unknown to American common law and jurisprudence in the

country’s first two centuries. Anonymity was first employed in federal prosecutions of

organized crime in New York in the 1980's. Although anonymous juries are unusual

since they are typically only empanelled in organized-crime cases, its use has spread

more recently to widely publicized cases, such as the federal prosecution of police

officers accused of beating Rodney King and the trial of those accused of the 1993

World Trade Center bombing.

In these cases, attorneys selected a jury from a panel of prospective jurors whose

names, addresses, ethnic backgrounds and religious affiliations remained unknown to

either side. This unorthodox procedure, designed to protect jurors from outside

influence and the fear of retaliation, has occasionally been employed in New York

federal courts since the trial of drug kingpin Leroy "Nicky" Barnes. Despite apparent

benefits, critics assail anonymous juries on the grounds that they are an infringement

of the sixth amendment guarantee of an impartial jury and because they present a

serious and unnecessary erosion of the presumption of innocence.

Since many attorneys believe trials are frequently won or lost during jury selection,

any procedure diminishing the role of counsel in the procedure necessitates close

scrutiny and criticism. Opponents of anonymous juries argue that the procedure

restricts meaningful voir dire, (questioning of the jury panel), and thereby undermines

the defendant's sixth amendment right to an impartial jury. Critics also claim that

jurors interpret their anonymity as proof of the defendant's criminal proclivity,

thereby subverting the presumption of innocence.

However, consistent with due process and the sixth amendment, the trial judge may

refuse to ask prospective jurors any questions not reasonably calculated to expose

biases or prejudices relevant to the case. Although addresses and group affiliations

may indicate significant potential for bias, attorneys do not have an unfettered right to

this information in every circumstance. Denying access to these facts may indeed

constrain an attorney's ability to assemble an ideal jury, but it violates no

constitutional right.

1. The primary purpose of the passage is to

Enumerate reasons why anonymous juries are unconstitutional

Discuss whether anonymous juries are an infringement of the sixth amendment

Identify a shortcoming in a scholarly approach to jurisprudence

Define the concept of anonymous juries and explore efforts taken over the last twenty

years to increase their use

Review strategies for ensuring that anonymous juries will not infringe on the

constitutional right to a fair trial of one’s peers

The best answer is B. The passage introduces the concept of anonymous juries and

goes on to discuss their constitutionality.

2. It can be inferred from the passage that a jurors ethnic background and religious

affiliation

background image

Is considered by defendants not to have a significant effect on the outcome of their

trials

Is considered by defendants to have a significant effect on the outcome of their trials

Would be unlikely to have a significant effect on the verdict of a trial

Is considered by attorneys likely to have a significant effect on the verdict of a trial

Is considered by attorneys unlikely to have a significant effect on the verdict of a trial

in a widely publicized case

The best answer is D. In paragraph three it states that many attorneys believe trials are

frequently won or lost during jury selection. The passage gives no information on

what defendant think about anonymous juries.

3. One function of the fourth paragraph of the passage is to

Qualify the extent to which a previously introduced viewpoint may be relevant

Expose the flaw in a criticism put forth in a previous paragraph

Introduce information that supports a theory put forth in a previous paragraph

Support an argument in favor of a given interpretation of a situation

Show the chain of reasoning that led to the conclusions of a specific study

The best answer is B. Critics of anonymous juries base their arguments on the fact

that these juries are unconstitutional. In the fourth paragraph, the author explains that

while anonymous juries may not be ideal, they are not unconstitutional.

4. Which of the following, if true, would ensure that anonymous juries are not an

erosion of the presumption of innocence?

Anonymous juries are used in all court cases, regardless of identity of the defendant.

Anonymous juries are used in all court cases involving previously convicted

defendants.

Anonymous juries are used in all court cases, involving never before convicted

defendants.

Anonymous juries are used in all widely publicized court cases, regardless of identity

of the defendant.

Anonymous juries are used in all widely publicized court cases, involving previously

convicted defendants.

The best answer is A. Anonymous juries are a potential erosion of the presumption of

innocence only because they are now used in cases in which there is a perceived

danger to the jurist, which presupposes a defendant capable of perpetrating a crime. If

anonymous juries were used in each and every case, they would no longer

differentiate between dangerous and harmless defendants.

background image

Alexander Calder was one of the most innovative and original American artists of the

twentieth century. Calder arrived in Paris in 1926 and devoted himself to a innovative

project comprised of animals made out of wire, scraps of cloth, wood, cork, labels,

bits of scrap metal and pieces of rubber that he called the Circus. During his

performances, Calder invented ways to simulate the flight of birds: “These are little

bits of white paper, with a hole and slight weight on each one, which flutter down

several variously coiled thin steel wires which I jiggle so that they flutter down like

doves.” The Circus was the laboratory of Calder’s work; in it he experimented with

new formulas and techniques. By 1930, Calder's Circus had developed into one of the

real successes of the Montparnasse art world attracting the attention of such renowned

artists as Fernand Leger and Joan Miro. Encouragement from the upper echelons of

the Parisian art scene undoubtedly led him to try more serious experiments in wire

sculptures.

Calder eventually becoming interested in the movement of objects, some of which he

motorized. In 1933, Calder completed Object with Red Discs, a sculpture he

described as a two-meter rod with a heavy sphere, suspended from the apex of a wire,

giving it a cantilever effect. It had five thin aluminum discs projected at right angels

from five wires, held in position by a spherical counterweight. With this new creation,

the idea of the mobile was born. In creating a work named Constellations in 1943,

Calder explored the plastic possibilities of mobiles; he used small pieces of wood,

which he shaped and sometimes painted. From this point on, Calder’s ambition

changed focus. He sought more challenging designs. One of Calder’s objectives was

to display objects in the air, giving the viewer the experience of finding new skies

filled with moving and colored constellations. Calder accomplished this in Acoustic

Ceiling (1954). Calder’s humor was evident in such works as Le Bougnat (1959) and

The Pagoda (1963). Later, Calder cut fantastic animals from sheet metal, creating La

Vache and Elephant (both 1970) and a mobile entitled Nervous Wreck (1976), which

represents the red skeleton of a fish.

Calder defined volume without mass and incorporated movement and time in art. His

inventions, which redefined certain basic principles of sculpture, have established him

as the most innovative sculptor of the twentieth century.

1. According to the passage, which of the following is an accurate statement about

Object with Red Discs?

It was the first mobile created by Calder.

It was one of the many mobiles without motors created by Calder.

It was one of the many motorized mobiles created by Calder.

It was the first motorized mobile created by Calder.

It was the first of the many mobiles without motors created by Calder.

The best answer is A. According to the passage, Object with Red Discs is Calder’s

first mobile. It states that Calder became interested in the movement of objects, some

of which he motorized, but there is no information given on whether this particular

sculpture was motorized.

2. According to the passage, all of the following are characteristic of Calder’s work

EXCEPT

Calder was known to infuse humor into some of his creation

background image

Calder suspended objects from each other

Calder motorized some of his creations

Calder used materials such as metal, cloth, wood, rubber, cork

Calder suspended glass from thin metal wires to create a cantilever effect

The best answer is E. The passage makes no mention of glass as one of the materials

Calder used.

3. The author’s attitude toward the mobiles of Alexander Calder is best described as

Hesitance

Detachment

Amusement

Admiration

Indifference

The best answer is D. The author presents only a positive criticism of Calder, stating

that he is the most innovative sculptor of the twentieth century.

4. It can be inferred from the passage that which of the following statement was true

of the Parisian art scene?

The work of Fernand Leger and Joan Miro was influenced by that of Alexander

Calder.

The work of Alexander Calder was influenced by that of Fernand Leger and Joan

Miro.

Fernand Leger and Joan Miro had earned success in the art world before Alexander

Calder.

Alexander Calder had earned success in the art world before Fernand Leger and Joan

Miro.

Calder’s Circus earned more accolades from the upper echelons of the Parisian art

scene than any other work in its time.

The best answer is C. According to the passage, Calder’s early work attracting the

attention of such renowned artists as Fernand Leger and Joan Miro. It can be inferred

that Leger and Miro were already famous when Calder was just starting out.

background image

Intuitively, intellectual skills and perceptual-motor skills seem very different because

perceptual-motor skills appear more primitive. Ontogenetically, perceptual-motor

skills develop before intellectual skills, or at least before most intellectual skills are

manifested. Phylogenetically, creatures "high on the evolutionary ladder" are more

obviously capable of intellectual skills than are creatures "lower down

Perceptual-motor skills also seem more closely tied to specific forms of expression.

Being a chess player does not mean one can only play with pieces of a certain size,

that one can only move pieces with one's right hand, and so on. By contrast, being a

violinist means one can play an instrument whose size occupies a fairly narrow range

and that one must play with a rather rigid assignment of functions to effectors

(bowing with the right hand, and fingering with the left). The seeming narrowness of

this perceptual-motor skill expression, contrasted with the seeming openness of

intellectual skill expression, seems to follow from intellectual skills having symbolic

outcomes and perceptual-motor skills having non-symbolic outcomes. Symbolic

outcomes need not be realized in specific ways and can rely on abstract rules. Non-

symbolic outcomes, by contrast, need more specific forms of realization and seem to

depend on restricted associations between stimuli and responses

Another difference between intellectual and perceptual-motor skills is that the two

kinds of skill seem to be represented in different parts of the brain. For example,

structures homologous to the optic tectum, a nucleus located on the dorsal surface of

the midbrain, have a common function in all vertebrates--coordinating visual,

auditory, and somatosensory information relevant to the control of orienting

movements of the eyes, ears, and head. Similarities in structure and function between

these and other brain areas associated with perceptual-motor behavior suggest that

mechanisms for control of perceptual-motor skills are both highly specialized and

conserved across species. In contrast, what distinguishes the human brain from the

brains of other species -- even closely related ones -- is the differential growth of

brain regions most strongly associated with intellectual skills, such as the association

areas of the cerebral cortex.

The contention that these areas serve intellectual functions is supported by a large

body of clinical and experimental literature. Together, these diverse sources of

information suggest that perceptual-motor and intellectual skills depend on distinct

brain circuits

1. The passage is chiefly concerned with

Presenting a new theory and describing a new method to test that theory

Suggesting an alternative to an outdated research method

Demonstrating that perceptual-motor skills are closely tied to specific forms of

expression

Arguing that two seemingly dissimilar skills are more alike than was previously

assumed

Presenting evidence on two dissimilar skills that resolves a contradiction

The best answer is D. The passage presents evidence to back the claims that

intellectual skills and perceptual-motor skills are more similar than was once believed.

background image

2. The author mentions the game of chess in paragraph two primarily in order to

Present an example of an intellectual skill the mastery of which is not closely tied to

specific forms of expression.

Present an example of an intellectual skill the mastery of which is closely tied to

specific forms of expression.

Present an example of a skill that is both an intellectual skill and a perceptual-motor

skill, the mastery of which is closely tied to specific forms of expression.

Present an example of a perceptual-motor skill the mastery of which is not closely tied

to specific forms of expression.

Present an example of a perceptual-motor skill the mastery of which is closely tied to

specific forms of expression.

The best answer is A. Chess is an intellectual skill the mastery of which is not closely

tied to specific forms of expression. In other words, once one has learned to play the

game, one can vary the way one plays, by using one’s left hand instead of one’s right,

for instance.

3. It can be inferred from the passage that the optic tectum

Functions similarly in animal and in plants

Functions similarly in vertebrates and invertebrates

Is located in a comparable area of the brains of humans and giraffes

Coordinates somatosensory moment in snakes

Has a much more sophisticated structure than the cerebral cortex

The best answer is C. According to the passage, the optic tectum occupies the same

area of the brain in all vertebrates (animals with a spinal column).

4. The passage provides support for which of the following statements

Creatures "high on the evolutionary ladder" are not less capable of perceptual-motor

skills than are creatures "lower down

Playing a violin is a perceptual-motor skill.

The shape and size of the cerebral cortex is what distinguishes the human brain from

the brains of other species

Literature is an intellectual activity that is understood using the cerebral cortex of area

of the brain.

Perceptual-motor and intellectual skills exploit the same brain circuits

The best answer is B. The author uses the violin as an example of a perceptual-motor

skill.

background image

Considerable debate exists in the self-perception literature over the impact of

positively biased self-perceptions on social and psychological functioning. Positively

based self-perceptions are those in which an individual has a more positive opinion of

himself than objective indicators warrant. One view suggests that positive perceptual

biases are characteristic of normal human thought across a variety of domains and

correlate positively with good mental and psychological health. Certain researchers

and clinicians have even proposed that by boosting self-concepts, symptoms of

depression and levels of aggression may be reduced.

Investigators on the other side of the debate maintain that when most positive self-

perceptions are compared to an objective criterion, they appear neither positively

biased nor adaptive. In fact, Baumeister, Smart, and Boden suggest that positively

biased self-concepts may have a ‘dark side’. They proposed that it is persons with

very positive self-views who are prone to be aggressive. As a result, building up

individuals' self-perceptions may serve only to increase levels of aggression rather

than curb them.

According to Baumeister et al., not all individuals with positive self-perceptions are

going to be interpersonally aggressive. Rather, individuals who are extremely positive

in their perceptions of themselves and their functioning are proposed to be the most

likely to become angry and potentially violent. The mechanism that triggers

aggressive behavior by these individuals has been suggested to be negative social

feedback that challenges their positive self-views. Such threats to positive self-esteem

give rise to anger and hostility.

If negative social information is encountered that challenges established positive self-

perceptions, Baumeister et al. propose that individuals must choose to either accept

the feedback and lower their self-perceptions or reject the feedback to maintain their

positive self-views. The chosen reaction then influences their subsequent affective

states and behavioral expressions. By accepting the external appraisals and adjusting

self-perceptions downward, dysphoric feelings and social withdrawal may result.

Conversely, the rejection of the validity of the unfavorable feedback results in

feelings of anger and resentment toward the source of the threat. Dodge and

colleagues demonstrated that children who interpret social cues as threatening direct

their anger and aggression at the peers who gave the negative evaluations. Anger

stemming from the receipt of social criticism is a way to deny the legitimacy of the

negative information. By directing hostile reactions toward the source of the negative

feedback, the influx of disconfirming information may end. Unless individuals react

against the self-esteem threat, they may be compelled to revise their self-concepts

negatively, in line with the information provided. By discounting the negative social

feedback, individuals can protect themselves from dysphoric feelings and maintain

their positively biased self-perceptions, but they may be setting themselves up to

become interpersonally aggressive

Although positively biased self-perceptions may place individuals at risk for negative

social feedback and subsequent increases in aggressive behavior, not all positive self-

concepts are suggested to be harmful. The relationship between positive self-

perceptions and aggression may depend on the degree of perceptual distortion (i.e.,

moderate vs. extreme distortion). Baumeister (1989) and Baumeister et al. (1996)

proposed that an optimal range of moderate bias might exist within which mental

background image

health is encouraged. Maladjustment in psychological and social functioning is

suggested to occur when the degree of bias of self-perceptions shifts from moderate to

extreme levels. Extremely negative and positive perceptual bias would be related to

different but equally harmful difficulties.

1. The primary purpose of this passage is to

Present two explanations of a phenomenon and reconcile the differences between

them

Discuss a plan for investigation of a phenomenon that is not yet fully understood

Challenge the validity of a theory by presenting evidence that the opposite is true in

some cases

Summarize two theories and suggest a third theory that overcomes the problems

encountered in the first two

Present evidence that resolves a contradiction

The best answer is C. The passage challenges the validity of the theory that positively

biased self-perception is closely correlated with normal human thought and good

mental health by showing how a very high self-perception may lead to violent

behavior.

2. According to the passage, which of the following is mentioned as a factor in

determining whether an individual with positively biased self-perception would

actually be likely to perform an act of violence?

The gap between what the individual thinks about himself and how good he really is

The gender of the individual

The anger level of the individual’s peers

The individual’s ability to ignore dysphoric feelings

The validity of the unfavorable feedback

The best answer is A. It is the distortion between how the person really is and how he

perceives himself that is the most important factor, as opposed to the level of self-

perception itself.

3. The passage discusses the likelihood of violence stemming from which of the

following types of individuals?

An individual with a moderately positive self-perception who receives negative

feedback from his peers

An individual with a highly positive self-perception who receives negative feedback

from his peers

An individual with a highly negative self-perception who receives positive feedback

from his peers

An individual with a highly negative self-perception who receives negative feedback

from his peers

An individual with a dark side who receives negative feedback from his peers

background image

The best answer is B. According to the passage, when negative social information is

encountered by a person with a highly elevated level of self-perception, it may lead to

violence.

4. One function of the fifth paragraph is to

State a conclusion about facts presented in an earlier paragraph

Show the chain of reasoning that led to the conclusions of a specific study

Qualify the extent to which a previously presented theory may apply

Introduce information that confirms an established theory

Provide examples that support a new theory

The best answer is C. Up to the fifth paragraph, the passage discusses the possibility

that individuals with highly positive self-perception are prone to performing violent

acts. Paragraph three qualifies this by stating that not all positive self-concepts are

suggested to be harmful.

background image

Gastrostomy tubes are commonly used to provide nutrition and hydration for patients

unwilling or unable to maintain an adequate oral intake. Among hospitalized patients

aged 65 years or older in the United States, the number undergoing placement of a

gastrostomy tube increased from 61000 in 1988 to 121000 in 1995. In 1990 and

1991, roughly one in every hundred hospitalized patients aged 85 years or older

received a gastrostomy tube.

The short-term mortality rates following gastrostomy placement are high. In a cohort

of more than 7000 American veterans who underwent placement of percutaneous

endoscopic gastrostomy tubes between 1990 and 1992, median survival was 7.5

months and 1-year mortality was 59%. Among Medicare beneficiaries receiving

gastrostomy tubes in 1991, 30-day and 1-year mortality was 24% and 63%,

respectively. Because tube insertion itself is only rarely associated with fatal

complications, the high short-term mortality clearly reflects a substantial underlying

co-morbidity in this population. Most patients receiving gastrostomy tubes have

advanced dementia, other types of severe neurological impairment, cancer, or

advanced failure of other internal organs.

The growing use of tube feeding in a population with limited life expectancy

inevitably raises the following question: Do physicians discuss the benefits and

burdens of tube feeding adequately with patients or surrogate decision-makers before

gastrostomy tubes are inserted? Assessing benefits and burdens is an integral part of

informed decision-making and should precede any elective life-sustaining

intervention. However, anecdotal observations and a recent interview study raise

serious questions about the quality of the informed consent process preceding the

insertion of gastrostomy tubes.

A small body of literature suggests that fully informed patients or their surrogates

might in fact decline permanent tube feeding at a higher-than-expected rate. For

example, in the study by Callahan et al, nearly half of the patients undergoing

gastrostomy placement (or their surrogates) reported that no alternatives had been

discussed before insertion of the tube. O'Brien and colleagues asked 379 mentally

competent nursing home residents if they would want a gastrostomy tube if they

became unable to eat because of permanent brain damage; only 33% expressed a

preference for tube feedings in this circumstance. In an interview study of 121

competent patients with amyotrophic lateral sclerosis, only 28% favored feeding by

gastrostomy

1. The passage is primarily concerned with

(A) the morbidity and mortality rates associated with the use of gastrostomy tubes

(B) the proliferation of the use of gastronomy tubes in patients aged 85 years or older

(C) whether physicians adequately discuss the benefits and burdens of tube feeding

with patients or surrogate decision-makers before gastrostomy tubes are inserted

(D) the growing number of patients undergoing placement of a gastrostomy tube

(E) the complications that often follow the insertion of a percutaneous endoscopic

gastrostomy tube

background image

The best answer is C. The question raised in the passage is not whether to use

gastrostomy tubes and what the dangers of using one might be, rather the passage

examines the issue of informed consent before the insertion of a tube.

2. According to the passage, the high mortality rate following gastrostomy found in

research cited in paragraph two is not necessarily a direct result of the placement of a

gastrostomy tube because

(A) tube insertion itself is only rarely associated with fatal complications

(B) 30-day and 1-year mortality was 24% and 63% respectively in 1991 among

Medicare beneficiaries

(C) only cases in which the percutaneous endoscopic gastrostomy tube was

incorrectly place were examined

(D) all the patients in the studies mentioned suffered from life threatening diseases

(E) most of the patients in the studies mentioned suffered from life threatening

diseases

The best answer is E. In paragraph two it is stated that most subject tested suffered

from one of a number of potentially fatal diseases. Hence, the mortality rate found in

patients that receiving a tube, was not necessarily due to the use of the tube, but to the

disease itself or to some other treatment.

3. It can be inferred from the passage that

(A) there are alternatives to the use of gastrostomy tubes to provide nutrition but not

hydration for patients unwilling or unable to maintain an adequate oral intake.

(B) there are alternatives to the use of gastrostomy tubes to provide hydration but not

nutrition for patients unwilling or unable to maintain an adequate oral intake.

(C) there are alternatives to the use of gastrostomy tubes to provide nutrition and

hydration for patients unwilling or unable to maintain an adequate oral intake.

(D) the alternatives to gastrostomy tubes do not provide adequate nutrition and

hydration

(E) the alternatives to gastrostomy tubes provide more adequate nutrition and

hydration than the gastrostomy tubes

The best answer is C. The passage suggests that if patients were offered alternatives

to gastrostomy tubes, some of them would choose them. We can infer from this that

alternatives exist.

4. The author uses the word “only” in line ???

[third line from the end]

most likely in

order to

(A) highlight the oddity of the decision of the patients

(B) emphasize the relatively low percentage of patients that would opt for a

gastrostomy tube if given the choice

(C) point out the limited value of inserting a gastrostomy tube

(D) distinguish the primary factor in the decision making process of brain damaged

patients

(E) single out a unique merit of gastrostomy tubes for brain damaged patients

The best answer is B. The passage draws attention to the fact that patients and their

surrogates are often not asked if they agree to the use of a gastrostomy tube.

O'Brien’s study indicates that a relatively small percentage – only 33% - stated that

background image

they would want the tube if they were unable to eat because of permanent brain

damage.

background image

A ragtag group of idealistic hackers scattered round the world has created software

and devised a revolutionary method for writing it that poses a direct threat to

Microsoft's revenue. Their programs are already running most of the Internet.

According to a survey by the British consultancy Netcraft, the Web server software

Apache is used by more than half of all websites. Furthermore, software named

Sendmail moves nearly every e-mail message across the Internet, while the BIND

program acts as a traffic cop for most of the global network, directing messages down

the right connections to their final destinations

The proven robustness of these programs is worry enough for Microsoft. To make

matters worse, all of them, and many others, are completely free in two senses: one

does not have to pay for them, and the "source code" in which they are written is

openly available. Additionally, one may modify the programs and even sell the result.

The "open source" movement is Microsoft's worst nightmare: a group of programmers

that it cannot out-compete because its members are not motivated by profit, and which

it cannot buy because it does not exist as a formal company.

In the vanguard of the open source movement is Linux, started in 1991 by a 21-year-

old Finn, Linus Torvalds, who wanted to write a free alternative to Unix, a popular

but costly operating system. Today Linux is used by an estimated 7 million people,

and the number is growing rapidly. One of Linux's advantages is that it runs on almost

any hardware, from multi-processor supercomputers down to Palm Pilots. It is

compact (it can fit on a floppy), highly efficient and very fast

Torvalds did not invent the idea of software that is doubly free but he has stumbled

upon and developed a crucially important Darwinian dynamic. In a commercial

software company, every program is carefully planned, and writing tasks are allotted

unilaterally by the project leader. Linux is different. It is designed as a series of

modules, and anyone can work on any of these interlocking elements. Whether one’s

work gets included in the final release depends on the consensus view of how good it

is--natural selection in action. The only reward anyone, even Torvalds, gets for this

work is kudos from fellow hackers. That is enough, it seems, to attract a flow of keen

recruits, typically computer science students or software engineers who code Linux on

the side.

Such purposive anarchy is made possible by the Internet. Trial versions of programs

can be downloaded, and comments sent back to the authors, wherever they are.

Programs frequently evolve on a daily basis. With the help of the Internet, the Linux

model exploits the ingenuity of hundreds of programmers and hundreds of thousands

of testers. It is a pool of creativity that Microsoft, with its huge resources, will never

be able to match.

1. The author mentions that a particular bit of software is included into the final

release of Linux only by consensus as an example of

(A) how writing tasks are allotted unilaterally by the project leader

(B) the modularity of Linux

(C) Linux’s unorthodox system of rewards

(D) the careful planning that goes into Linux software

(E) the process of natural selection at work in the field of computer programming

background image

The best answer is E. The author describes the workings of Linux as having a

‘Darwinian dynamic’ – it works on the principle of natural selection. Only the fittest

software is used.

2. It can be inferred from the passage that if Linux existed as a formal company

(A) it would be used by more than the estimated 7 million people that currently use it

(B) its software would be more costly than Unix

(C) its software would still be less costly than Unix

(D) Microsoft would try to acquire it

(E) it could compete with Microsoft

The best answer is D. According to the passage, Microsoft cannot buy Linux because

it does not exist as a formal company.

3. According to the passage, programmers are motivated to create software for Linux

I. to earn accolades

II. for pecuniary gain

III. to learn how to successfully hack software

(A) I only

(B) II only

(C) I and II only

(D) II and III only

(E) I, II and III

The best answer is A. The author states that the only reward anyone gets for work on

Linux is kudos, or accolades, from fellow hackers.

4. The passage mentions each of the following as factors contributing to the success

of groups creating open source software EXCEPT

(A) the groups use the Internet to gather together hundreds of thousands of testers

(B) the groups do not exist as formal companies

(C) the products are designed to work on Palm Pilots

(D) the groups are not motivated by profit

(E) the programs evolve frequently

The best answer is C. The author mentions Palm Pilots only in relation to the Linux

program. No information is given on whether other open source software is versatile

enough to run on Palm Pilots.

The cyclic rise and fall in population size that has been observed in animal and pre-

modern human populations reflects sequential phases of population growth and

decline. Disregarding the possible selective influences of migration, these cyclic

movements must ultimately be accounted for in terms of the range of variation in

fertility and mortality.

No downward trend in mortality is apparent in any country before the middle of the

eighteenth century, about the same time that population growth began to demonstrate

an exponential curve. The initial period of sustained population growth in nearly

background image

every country for which reliable data are available corresponds with at least two

decisive changes in the death rate. First, the fluctuations in mortality became less

frequent and less drastic. Second, the initial, slow--sometimes imperceptible--decline

in mortality gradually gained momentum and eventually stabilized at relatively low

levels in the twentieth century. Thus, steady increases in life expectancy,

progressively diminishing death rates and more stable and predictable mortality

patterns have accompanied the persistent increments in world population.

In England and Wales, Japan, Ceylon and Chile an exponential pattern of population

growth has accompanied the downward trend in mortality. In England and Wales,

where the transition from high to low vital rates occurred over two centuries, the

exponential growth curve was attenuated only after fertility fell and approached the

low level of mortality; this pattern was less apparent for Japan, where an accelerated

transition occurred over several decades. Although data concerning the relative effects

of mortality and fertility on population growth are incomplete for the early transitional

period, it seems likely that a significant though temporary increase in fertility may

have added momentum to the population explosion set off by steady improvements in

survivorship. The influence of fertility is particularly apparent in the rapid population

growth of currently developing nations that have not yet completed their transitions,

for example, Chile and Ceylon. In most of these developing countries, the death rate

has declined rapidly in recent years, especially since World War II, and the birth rate

has remained high with minor fluctuations. This sudden widening of the demographic

gap has produced unprecedented high rates of population growth.

1. Which of the following best describes the content of the passage?

(A) A discussion of how recently developed methods of monitoring population

growth differ from older methods

(B) A description of some of the factors effecting cyclic changes in population

(C) A chronology of the development of different methods for monitoring population

growth

(D) A proposal for improving the accuracy of current methods to monitor fluctuations

in population

(E) An argument concerning the nature of the exponential pattern of population

growth has accompanied the downward trend in mortality

The best answer is B. (A) is incorrect because no recently developed methods are

mentioned in the passage. (C) is wrong because the information given is not

chronological. (D) and (E) are incorrect because the passage neither proposes nor

argues anything.

2. According to the passage, the fertility rate

(A) in Japan has remained constant over the last several decades

(B) in England and Wales has remained constant over the last two centuries

(C) fell at a similar rate in England, Wales and Japan

(D) fell at a faster rate in Japan than in England and Wales

(E) fell at a slower rate in Japan than in England and Wales

background image

The best answer is D. According to the passage the transition from a high fertility

rate to a low one occurred over two centuries in England and Wales. The fertility fell

in Japan over a period of several decades.

3. According to the passage, population increase in currently developing nations is a

result of

(A) the recent decline in death rate, especially over the last 50 years, and a birth rate

that is consistently high with minor fluctuations.

(B) the recent decline in birth rate, especially over the last 50 years, and a death rate

that is consistently high with minor fluctuations.

(C) the recent decline in death rate, especially over the last 150 years, and a birth rate

that is consistently high with minor fluctuations.

(D) the recent decline in birth rate, especially over the last 150 years, and a death rate

that is consistently high with minor fluctuations.

(E) the recent rise in death rate, especially over the last 50 years, accompanied by a

similar rise in birth rate.

The best answer is A. According to the passage, the death rate in currently

developing nations has declined rapidly since World War II, while the birth rate has

remained high with minor fluctuations.

4. It can be inferred from the passage that the first downward mortality trend

(A) followed great fluctuations in fertility rates

(B) followed great fluctuations in death rates

(C) appeared after World War II

(D) appeared before World War II

(E) appeared around 1750

The best answer is E. According to the passage, no downward trend in mortality was

noticeable before the middle of the eighteenth century.

background image

Researchers criticize the transaction cost economics (TCE) paradigm for over-

generalizing the assumption of opportunism as part of human nature. However, some

suggest that individualists have a higher opportunistic propensity in intra-group

transactions, and collectivists in inter-group transactions. This cultural specification of

opportunism helps TCE to accommodate more effectively some criticisms and more

realistically deal with problems of economic organization in today's global economy.

Transaction cost economics was first proposed by Coase and later popularized by

Williamson. One of its key building blocks is the assumption of opportunism,

because individuals, as Williamson claims, "will not reliably self-enforce promises

but will defect from the letter and spirit of an agreement when it suits their purposes".

Since, evidently, not all economic players are likely to be opportunistic, such a

reliance on the assumption of opportunism has resulted in a torrent of criticisms,

calling it dangerous, unhealthy, bad for practice, and an ethereal hand for

organizational researchers. A primary reason TCE has provoked such a debate is

precisely because it is centered on the assumption of opportunism, which touches on a

fundamental question of human nature.

To further develop this paradigm it is necessary to respond to the criticisms by

clarifying and strengthening this important assumption. To be sure, TCE scholars

never assumed that all (or most) individuals are likely to be opportunistic all (or most

of) the time - a clearly indefensible position. Instead, most individuals are assumed to

be "engaged in business-as-usual, with little or no thought to opportunism, most of the

time". However, TCE suggests that it is the inability to differentiate opportunists,

who may be a minority, from non-opportunists ex ante that necessitates the

assumption of opportunism. TCE’s critics, on the other hand, do not suggest that

opportunism does not exist; rather, they caution against an over-reliance on the

opportunism assumption because it may not be realistic to hold this assumption

constant across individuals and organizations around the world. In order to make

further theoretical progress, researchers must tackle the harder and more interesting

issues of what kinds of individuals are likely to be opportunists, under what

circumstances, and to what extent. Such an improved understanding of opportunism is

important in today' s increasingly global economy, in which economic players from

different backgrounds routinely interact with each other.

1. In the passage, the author is primarily concerned with doing which of the

following?

(A) Comparing two different approaches to a problem

(B) Present a criticism of an assumption and suggesting how to clarify it

(C) Describing a problem and proposing a solution

(D) Presenting data and drawing conclusions from the data

(E) Comparing two different analyses of a current situation

The best answer is B. The passage presents criticism that TCE’s assumption of

opportunism has provoked and suggests what research should be carried out to resolve

the issue.

2. It can be inferred from the passage that which of the following is true of TCE?

(A) All economic players are likely to be opportunistic

(B) The majority of economic players are opportunistic

background image

(C) The majority of human beings are opportunistic

(D) Not all opportunists are opportunistic under the same circumstances

(E) TCE has provoked such a debate because it is centered on the assumption of

fundamentalism

The best answer is D. According to the passage, TCE advocates claim that

individualists have a higher opportunistic propensity in intra-group transactions, and

collectivists in inter-group transactions. Thus, it can be inferred that not all

opportunists act in an opportunistic fashion under the same circumstances

3. Which of the following would most logically be the topic of the paragraph

immediately following the passage?

(A) Specific ways to determine what kinds of individuals are likely to be

opportunists, under what circumstances, and to what extent

(B) The contributions of TCE to economic theory

(C) Ways in which opportunists take advantage of unsuspecting business men

(D) Nontraditional methods of testing individuals to find out if they are opportunists

(E) The centrality of TSE to the position of opportunists in the business world

The best answer is A. In the last paragraph of the passage it says that in order to make

further theoretical progress researchers must tackle the issue of what kinds of

individuals are likely to be opportunists, under what circumstances, and to what

extent. It is logical that the following paragraph would do so.

4. The quotation in line ???

["will not reliably self-enforce….]

is most probably used

to

(A) counter a position that the author of the passage believes is correct

(B) counter a position that the author of the passage believes is incorrect

(C) elucidate a term

(D) point out a paradox

(E) present a historical maxim

The best answer is C. The quotation is most likely used to explain what is meant by

the term opportunism.

background image

The mystery of cloud formation is as ancient as mankind. Throughout history, man

prayed for rain and feared snow and lightning. Only in the last few decades, was man

able to experiment and answer many questions as to how clouds are formed. Lately, it

has seemed that no almost queries were left in that field. However, a recent look at old

experimental data is threatening to overturn a longstanding theory about how water

droplets freeze within clouds

Suspended water droplets can remain liquid even at temperatures far below the

normal freezing point. Data collected in recent years show that clouds as cold as –

37.5°C can still contain many liquid droplets of water. Such droplets freeze solid

almost instantly if they bump into each other or are otherwise disturbed Most

scientists have long assumed that a tiny globule of pure, liquid water, when disturbed,

begins to freeze around an icy seed that suddenly forms inside it. According to this

scenario, the time needed to freeze a given volume of water dispersed into a fine mist

is independent of the size of the individual droplets because the formation of a seed

particle is a chance event

However, the results of recent laboratory experiments, when combined with

information gathered from tests conducted as many as 30 years ago, do not support

this scenario. Together, the data indicate that the time needed to freeze a given

volume of liquid water varies drastically according to droplet size. This extreme

variation makes sense if freezing begins at the surface of the drops, not at the cores.

Dividing a given volume of water into a large number of small droplets yields more

total surface area than if the volume is split into a small number of large drops. The

freezing rate would then depend on the surface area

The laws of thermodynamics also argue against ice nuclei forming inside liquid

droplets. When water molecules begin to assemble into ice crystals, they release large

amounts of latent heat. If that process had occurred in the center of a liquid droplet,

the heat would have remained trapped within the globule, slowing the freezing

process. But if crystallization begins at the droplet's surface, latent heat can more

easily transfer to the surrounding air. In this case, the droplets are so cold that heat

released internally as crystallization proceeds probably would not melt the developing

ice

Familiar as clouds are, the behavior of their constituent droplets remains only partly

understood. Lightning, rainfall, and other meteorological phenomena vary with the

ratio of water droplets and ice particles in clouds. Linking freezing rates of clouds to

those atmospheric and other climate processes is one of the most unreliable areas in

current climate simulations.

1. According to the passage, if a given volume of water is divided into a large

number of small droplets, as opposed to a smaller number or large droplets, it will

freeze quicker. This indicates that

(A) a given volume of water yields more droplets when below freezing

(B) a given volume of water yields more droplets when above freezing

(C) freezing begins simultaneously both at the cores and at the surface of the drops

(D) freezing begins not at the surface of the drops, but at the cores

(E) freezing begins not at the cores of the drops, but at the surface

background image

The best answer is E. If the relatively large amount of surface area provided by small

droplets influences the time it takes to freeze, then freezing must begin on the surface

of the droplets.

2. According to the passage, all of the following are characteristic of cloud droplets

EXCEPT

(A) they periodically bump into each other

(B) the time needed to freeze a given volume of water dispersed into a fine mist is

dependent on the size of the individual droplets

(C) can remain liquid even at temperatures far below the normal freezing point

(D) they are only partly understood by scientists

(E) they appear in the same amount in clouds involved in various meteorological

phenomena

The best answer is E. According to the passage, the ratio of water droplets and ice

particles in clouds varies in lightning, rainfall, and other meteorological phenomena

3. The passage suggests that many questions in the field of meteorology

(A) have been answered

(B) have no answers

(C) have yet to be answered

(D) have been answered incorrectly

(E) are uninteresting to scientists today

The best answer is A. The author states that it has lately seemed that no almost

queries were left in meteorology.

4. The primary purpose of the passage is to

(A) present several explanations for a well-known fact

(B) argue in favor of a long-standing theory

(C) discuss the implications of a new research finding

(D) present new evidence that overturns a long-standing theory

(E) question the methodology used in a study.

The best answer is D. The passage presents new evidence on how water droplets

freeze which overturns a long-standing theory that claims that droplets freeze from

their center.

background image

To unravel the extent to which Shakespeare is accorded cultural capital in India, two

examples from popular culture will be examined - one from an Air India

advertisement and another from a Hollywood film in the making that casts

‘Bollywood’ star Hrithik Roshan as an Indian king in Rajasthan, looking to avenge

the murder of his father.

The Air India advertisement, found in a 1975 American magazine, features

Shakespeare with a beautiful Indian woman clad in a sari. Focused from shoulders

up, the black and white image reproduces a typical studio shot of a married couple.

The top of the ad reads, We go out of our way to please you. We also go to London

every day." This is followed by the contact reference to Air India. Presented in the

image of a couple, the ad naturalizes the union of the colonized Indian woman and

Shakespeare (the most authoritative representative of the culture of the colonizer) and

thrusts them into the global realm of tourism, art, and commerce to symbolize the

union of East and West, with Shakespeare representing the means of transcending the

distance between India and England. To make it more enticing, the slogan that

follows the main caption of the ad is also somewhat sexualized: "We (Air India) work

all day to make your night with us a dream." The advertisement creates a spectacle

that reproduces Shakespeare in the image of the authoritative imperial traveler to the

exotic and mysterious land, India, represented through the image of the exotic Indian

woman. In so doing, the advertisement at once obscures the gendered, racial, and

colonial dynamics that complicate such an image and legitimizes the discourse around

Shakespeare as a harbinger of cultural authority

My second example is a Hollywood film in the making that plans to cast Indian

Bollywood star Hrithik Roshan in the role of a Rajput prince who avenges his father's

murder in a "chilling thriller." To generate publicity for the film, the filmmakers

proposed to market Roshan as an "Indian Hamlet." While the producer believes that

Hrithik is the ideal candidate for the role and places him "somewhere between Tom

Cruise and Antonio Banderas," he recognizes that Hrithik is a new name in

Hollywood. Therefore, he says, "we will have to sell him through the two known

names--Hamlet and Shakespeare". In doing so, the Hollywood producer assumes the

far-reaching influence of and appreciation for Shakespeare. At the same time, as a

film that will be shot in India with some scenes in England, it conjures up the image

of exoticism that continues to seize the West's imagination.

Such images not only keep alive Shakespeare’s insidious influence in India, they

further perpetuate the discourse about the bard's cultural superiority, which continues

through government-sponsored agencies, the continuing presence of Shakespeare

studies in education, and through theatre groups and touring companies performing

locally and visiting from abroad.

1. According to the passage, Hrithik has to be marketed as India’s Hamlet because

(A) his name is unknown to American audiences

(B) American audiences confuse him with Tom Cruise and Antonio Banderas

(C) the movie’s Hollywood producer assumes the far-reaching influence of and

appreciation for Shakespeare

(D) the film will be shot in India and England

(E) Hrithik is the ideal candidate for the role

background image

The best answer is A. The author states that since Hrithik is a new name to American

audiences he will have to be sold through two familiar names, Hamlet and

Shakespeare.

2. The author’s attitude toward the cultural capital accorded to Shakespeare in India

is best described as one of

(A) indifference

(B) hesitance

(C) disapproval

(D) amusement

(E) neutrality

The best answer is C. In the last paragraph, the author refers to Shakespeare’s

influence as insidious.

3. In can be inferred from the passage that more people in the U.S. are likely to pay to

see the Hollywood movie mentioned in the passage if the actors are

(A) Indian

(B) British

(C) Classically trained

(D) familiar to the audience

(E) excellent at portraying their roles

The best answer is D. No information is given in the passage on how the factors

mentioned in choices A, B, C, and E might influence a movie-goer.

4. In the last paragraph, the author is primarily concerned with

(A) correcting an error occurring in one of the works under review

(B) citing evidence to support a view of Shakespeare in India

(C) drawing conclusions on the basis of evidence presented in the first three

paragraphs

(D) summarizing the arguments about Shakespeare presented in the first three

paragraphs

(E) refuting the view of Shakespeare’s influence in India presented in the previous

paragraph

The best answer is C. In the last paragraph, the author draws conclusions on the

effects of the examples presented earlier in the passage.

background image

The relationship between science and religion, and even the one between skepticism

and religion, is warming up. At least, that is the feeling one gets from a cursory look

at recent happenings, including the proliferation of books and articles in popular

magazines about science "finding" God.

Thus, the time is ripe for a skeptical analysis of the subject, which seems muddled by

two basic sources of confusion: the need to separate logical/philosophical arguments

from those that are either pragmatic or concern freedom of speech; the need to

acknowledge that there are many more possible positions on the science and religion

question than are usually considered, and that a thorough understanding of the whole

gamut is necessary to make any progress.

The relationship between science and religion is a legitimate area of philosophical

inquiry that must be informed by both theology and science Discussions about

science and religion, especially in the United States, carry practical consequences that

do not affect both in an equal manner. Discussing science and religion has

repercussions on the cherished value of freedom of speech for scientists, skeptics, and

religionists

Attacks on religion are considered politically incorrect--the remarks by Minnesota

Governor Jesse Ventura resulted in his popularity dropping 28 percent overnight.

Scientists are especially aware of the fact that their research funding depends almost

entirely on public financing through various federal agencies such as the National

Science Foundation and the National Institutes of Health. Since federal funding is

controlled by politicians, who in turn have a tendency to respond to every nuance of

their constituency as gauged by the latest poll, it follows that no matter what your

opinion as a scientist on matters of the spirit, it is wiser to stick to your job and avoid

upsetting your benefactor

This is all the more so because of two other things we know about scientists: the

overwhelming majority of them do not believe in a personal God (about 60% of

general scientists and a staggering 93% of top scientists), and the reason they become

scientists is to pursue questions for which science is a particularly good tool. Most of

these questions are rather more mundane than the existence of God

The result of this odd mix is that while most prominent scientists do not believe in a

personal God because of their understanding of science and of its implications, they

must come out in public with conciliatory statements to the effect that there is no

possible contradiction between the two.

1. The author mentions Governor Jesse Ventura (lines ???) most likely in order to

(A) prove that the Governor does not believe in a personal God

(B) show that the Governor believes in a personal God and therefore does not believe

that various federal agencies such as the National Science Foundation and the

National Institutes of Health should back scientific enquiry

(C) show that the public is unlikely to finance a politician who denounces religion

publicly

(D) show that the public is unlikely to support a public figure who denounces religion

publicly

(E) show that the public is likely to back a public figure who denounces religion

publicly

background image

The best answer is D. The author uses the governor’s case to show that if a public

figure speaks out against religion that person would likely lose the support of the

public

2. It can be inferred from the passage that the author believes that the questions asked

by scientists are

(A) more ordinary than questions about the existence of God

(B) more extraordinary than questions about the existence of God

(C) more popular than those asked by theologians

(D) more popular than those asked by politicians

(E) less extraordinary than those asked by politicians

The best answer is A. The author states that science delves into questions that are

more mundane than the existence of God.

3. The author implies that which of the following will occur if a scientist publicly

declares that he does not believe in a personal God?

(A) That scientist will likely lose his job.

(B) That scientist will likely lose all support.

(C) That scientist will likely lose federal support.

(D) That scientist will likely lose all funding.

(E) That scientist will likely lose federal funding.

The best answer is E. According to the passage, a scientist that speaks out against a

belief in God would likely lose federal funding because that funding is decided upon

by politicians who do not want to lose the support of their voters.

4. The passage as a whole can be characterized as which of the following?

(A) A description of an attitudinal change

(B) A discussion of an analytical defect

(C) A look at the interrelationship of two fields of inquiry

(D) an argument in favor of revising a view

(E) an evaluation of a scholarly study

The best answer is C. The passage examines the relationship between science and

religion.

background image

The Technology-Related Assistance for Individuals with Disabilities Act of 1988
defines assistive technology devices as any item, piece of equipment, or product
system, whether acquired commercially, modified, or customized, that is used to
increase, maintain, or improve functional capabilities of individuals with disabilities.
The promise offered by this law is that a student with a disability will be offered any
assistive devices and services necessary to enhance his or her educational experience.

Although there is general agreement that technology is educationally beneficial, the
research results have not strengthened the case for its impact on teaching children
with disabilities. However, Edwards not only encourages an expanded use of
technology in education, but also offer several reasons why its continued use is
justified. For example, he believes that technologically based methods have promoted
several motivational strategies such as gaining learner attention, getting learners to
create their own technologically based products, and empowering learners to take
control of their own learning. He also contends that technology can facilitate unique
learning environments, make traditional learning environments more powerful and
more effective by linking learners to information sources, help learners visualize
problems and solutions, and connect learners to learning tools

While most agree that technology has obvious appeal for improving educational
outcomes for students, teachers must ensure that it accommodates the specialized
needs of students with disabilities. The diversified needs of students with various
disabilities (e.g. behavior disorders, mental retardation, blind or vision impairment,
deafness and so on) require different technological resources and applications. For
example, assistive technology devices have been designed to bypass the need to type
for learners with physical disabilities that effect their hands

One should consider the following factors when selecting and using assistive devices.
The user should ensure that the assistive device can be integrated into the student's
instructional program, can endure rapid technological changes and, if warranted, can
be upgraded, and can be easily maintained and that technical support, repair or
maintenance is readily available. Additionally, school districts should ensure that
teachers receive the necessary training and support for employing assistive devices

background image

and services so that teachers can provide students with the necessary motivation to
use assistive devices

background image

'.

&

&

&

+.,

&

)

)

+6,

&

)

)

+!,

&

)

&

+7,

&

)

&

+ ,

&

)

#

)

.'.

&

&

&

&

&

)

&

&

)

'

8'.

&

&

)

&

)

+.,#

&

)

0

&

5

)

0 &

)

)

)

'

+6, #

&

)

)

)

&

*

)

+!, #

&

)

&

)

+7,#

&

)

0

&'

+ ,#

&

)

)

&

)

)

) '

#

)

7'

&

0 &

&

&

)

)

'

background image

9'.

&

&

&

)

&

)

#

& *-

.

7

)

.

DD :! ;#

+.,

)

)

+6,

3

)

)

)

+!,

3

)

2

)

)

+7,

3

)

)

)

+ ,

3

)

)

#

)

!'#

&

&

&

&

)

&

)

)

'

<' .

&

&

&

)

)

&>

B &

E 0

+.,

+6,

+!,

+7,

+ ,

#

)

!'

&

)

&

'

'

background image

Economists have defined four stages societies pass through based on demographics,
economics, agricultural productivity, and technological advancements. The first stage
is a traditional society, characterized by low population density and low economic and
population growth. In this society, which may be a country or a region, high birthrates
match high death rates, while primitive technology contributes to low income and low
living standards.

The second stage is a developing stage, occurring when a society's technological
advancements result in sustainable agricultural production and plant and animal
domestication. The result is a more plentiful food supply, which helps increase
population growth mainly by slowing death rates. When population and food
production growth are combined with industrialization and urbanization at this stage,
the result is environmental exploitation and degradation present in many developing
societies

Increases in agricultural productivity and production bring economic surpluses that
allow growth in capital and per capita income and the third stage, the developed
society, is born. Birthrates fall faster than death rates as the roles of women change
and developments in birth-control methods allow adults to choose the number of
children they desire

Finally, stage four, the mature society, sees notable technological change evolve
beyond agriculture (particularly in medicine and public health), making death rates
decline further. While many technological breakthroughs in developing societies
come from innovative laypersons, breakthroughs in developed societies tend to
require scarce, highly trained, experienced, and costly technicians and scientists. By
the time the mature society develops, the most readily accessible raw materials have
been exploited. Obsolescence of current technology requires investment in
maintenance rather than in new technologies. Increase in productivity of service
activities, which grow in importance, becomes more difficult than increases in
agricultural and manufacturing productivity. Some developed regions choose to
sacrifice some economic growth for equity. Thus, while productivity and income
continue to rise, the rate of these increases slows.

background image

Rapid agricultural productivity gains continue in developed societies as investments
in education and science made in the development stage produce long-term payoffs
and as urbanization and industrialization lead to an exodus of agricultural labor. At
the same time, slowing rates of income growth and population growth slow down the
growth in demand for food. Food self-sufficiency increases in some countries after
falling in the development stage. However, agricultural trade typically grows, as more
affluent consumers demand a variety of foods from around the world

Many developed countries have recently entered or will soon enter the fourth stage--
the mature society--the future society of the world's inhabitants. It is a long-held view
that global population growth will more or less stabilize; recent evidence, however
presents a strong case for negative global population growth as the seminal attribute
of the mature society

'#

&

+.,

&

+6,

2

&&

)

+!,

)

+7,

&

+ ,

&

)

(

&

#

)

!'#

&

)

&

&

'

8'

)

&

)

&

+.,

&

+6,

&

+!,

&

+7,

&

background image

+ ,

&

#

)

7'.

&

&

)

&

&'

)

)

&

'

9'.

&

&

& 2

&

&

)

)

+.,

+6,

+!,

+7,

B

1

+ ,

&

#

)

6'#

&

"

&

F'

4. According to recent evidence, negative global population growth is considered

+.,

)

+6,

+!,

&

)

+7,

&

)

+ ,

&

The best answer is C. The author states that negative global population is “the
seminal attribute of the mature society ”

background image

According to the United Nations, the total fertility rate - the number of children a
woman may be expected to bear during her lifetime - has fallen in every region of the
world since 1950. From an average of nearly six children per woman in the 1950s,
total fertility rate fell to three children in Latin America, 3.4 in India, and 3.5 in other
parts of Asia by the early to mid-1990s. The only major exception to this sustained
downtrend is in North America, where the recent increase in total fertility rate appears
to be a transitory phenomenon associated with immigration and a large number of
baby-boom women deciding to have children relatively late in their lives

As significant as declining total fertility rates worldwide is the fact that, from 1990 to
1995, the rates in Europe, China, and North America were below the 2.1 average
children per woman needed to sustain population worldwide over the long run. The
United Nations' medium population projection of 2.1 children after 2040 is widely
used as a demographic forecast, but it unrealistically assumes that this rate will be the
same in both developed and developing countries. Many researchers, such as
Wolfgang Lutz of the International Institute for Applied Systems Analysis, do not
support the UN assertion that fertility would increase to replacement level in
developed countries. Lutz and others cite evidence pointing toward low fertility,
noting contraception, declining marriage rates, high divorce rates, increasing
independence and career orientation of women, materialism, and consumerism.

These factors, together with increasing demands and personal expectations for

attention, time, and also money to be given to children, are likely to result in fewer
couples having more than one or two children and an increasing number of childless
women," Lutz and his colleagues write in The Future Population of the World. The
United Nations has a second scenario - the low/medium scenario - that presumes
fertility averaging 1.9 children per woman for all regions by 2025. This scenario may
be as unrealistic as the medium population scenario. The low/medium scenario may
underestimate future total fertility rates in developing countries, just as the UN
medium scenario may overestimate future total fertility rate in developed countries

The low/medium scenario projects a peak world population of 7.9 billion people in
2050, declining to 6.4 billion by 2150. The medium scenario projects a peak world
population of just less than 11 billion by approximately 2200. Most other projections,

background image

however, predict peak global population in less than a century, followed by negative
population growth
These 1998 UN population figures were revised in 2000, and the new estimates,
though tentative, indicate population trends even lower than the 1998 predictions. The
low/medium scenario is comparable to what the United Nations now calls the low
variant for worldwide populations. Given the low variant, the United Nations predicts
that world population in 2050 will be 7.8 billion--slightly less than the 7.9 billion
projected under the low/medium scenario. It is predicted that the United Nations will
continue to revise population trends downward and that negative population growth
will occur even sooner than 2150. However, because the 2000 population data remain
tentative and do not extend to 2150, it is preferable to continue to rely on the 1998
data.

'4

&

)

& >

+.,

&

B

%

&

)

+6,

&

B

%

&

)

+!,

&

B

%

&

)

+7,

&

B

%

&

)

+ ,

&

B

%

&

)

&

)

.

#

)

'%

)

)

&

)

&

&

)

.

& '

8'

)

&

+.,

)

%

*

.

?

+6,

)

.

?

background image

+!,

)

1

.

?

+7,

)

1

.

&

+ ,

)

1

.

#

)

6' .

&

&

&

(

1

.

)

.

'

9'.

&

) 4

& &E 2

&

0

>

+.,

'

+6,

&

+!,

&

)

) )

'

+7, E

'

+ , #

)

)

)

'

#

)

.'.

&

& 4

& &E 2

&

)

'

<'.

&

B

1

5

&

G

+.,

0

(

8? ?

0

(

88??

+6,

0

(

8? ?

0

(

8

?

+!,

0

(

8

?

0

(

8? ?

+7,

0

(

8

?

0

(

88??

+ ,

0

(

88??

0

(

8

?

#

)

.'#

G

&

H

' )

H

(

8? ?'#

&

H

)

H )

(

88??'

background image

The racial identity of South Asians has long been a subject of some controversy in the
United States. In the early years of the twentieth century, when whiteness, or African
ancestry, was a prerequisite for naturalization, American courts vacillated on the
question of whether Asian Indians were white or not. In contrast to Mexicans and
Armenians, who were deemed white for the purposes of citizenship acquisition, and
Japanese, Chinese, and Filipino applicants who were not, the verdict on the racial
classification of Indians changed from case to case. American uncertainty over South
Asian racial identity has also been mirrored in the Census Bureau's frequent changes
in its classification of this group. Over the course of the last century, respondents of
South Asian origin have been classified variously as `Hindu', `White', `Other', and
`Asian'.

South Asian newcomers are not alone, however, in confronting an American racial
landscape that at first seems to have no clear place for them. Not only does the
diversity of the United States' contemporary immigrant pool ensure a steady influx of
people who do not fit easily into the traditional black/white dichotomy, but in the past
as well, immigrants tested, stretched and molded the nation's conceptions of racial
categories. As Ignatiev has shown, Irish immigrants were not considered white until
well after their arrival in the United States, and this was true of other European groups
as well. Similarly, Chinese, Japanese, Korean, and Filipino Americans were not
always considered to constitute a pan-ethnic Asian race.

But unlike the Irish who have already become white, or the Chinese and Japanese
who are now Asian, the racial classification of South Asians in the United States is
still in flux. Although they now seem firmly ensconced in the census `Asian' category,
this is a recent development and one that came about only after considerable debate.
Moreover, several writers have described an uneasy alliance between South Asians
and East Asians under the pan-ethnic `Asian' rubric. Finally, other Americans seem
unsure as to the racial status of these immigrants. F. James Davis finds evidence that
some blacks consider Indians to be black as well, and Rosemary Marangoly George
reports a widespread concern among Indian Americans in California over being taken
for Mexican or black. More broadly, Nazli Kibria maintains that South Asians are
seen as `ambiguous non-whites' in the United States

background image


Defined as `the socio-historical process by which racial categories are created,
inhabited, transformed, and destroyed', racial formation is both a macro-level process
and the culmination of myriad individual encounters. Given their inchoate racial
status, South Asian Americans may offer unusual insight into this process of racial
formation.

'#

&

+.,

*

)

&

+6,

)&

*

B

%

+!,

)

B

%

+7,

(

)

%

.

B

%

+ ,

%

.

B

%

#

)

'!

.

7

&

'#

&

)&

*

)

%

.

B'

%'

8'#

0

+.,

()

&

)

+6,

&&

)

+!,

*.

+7, &&

%

.

(

+ ,

)

&

!

I

A

;

#

)

.'-

&

)

)

( '#

&

(

)

() '

background image

9'#

&

&&

)

&

&

)

2

>

+., .6

+6,.C

+!,.I

+7,. .

+ , . .

.

#

)

6'6

.

) &

2

'! 2

&

.

'

<'.

&

&

>

+.,

)

'

+6,

*

'

+!,

&

%

.

'

+7,

&

'

+ ,

'

#

)

6'

&

*

)

&

'

background image

In parallel with advances in drug therapy, anesthesia, and surgery, there has been a

shift in bioethics from a paternalistic ethic governed by doctors to one based on the

patient's autonomy and integrity. The notion of consent to medical procedures and

treatment is a reflection of this, and in many countries, this consent is now established

in law.


The terms "informed" and "patient's consent" were perhaps first coupled in the 1957
case of Salgo versus Leland Stanford Jr. University Board of Trustees. The plaintiff,
paralyzed after myelography, had not been informed by his doctor that paralysis was a
possible risk of this procedure. He won although the doctor had committed no mistake
because the doctor failed in his duty of disclosure. The court found that if the patient
had been properly informed he would have refused myelography.

From a legal perspective, any consent, if it is to be valid, has to meet three
independent preconditions, all of which have to be taken into account simultaneously.
Consent may be deemed invalid if it has been obtained by deception or coercion; if it
does not comply with formal procedures; or if the person lacks the capacity to consent
by virtue of mental illness. The weaker a patient's personal competence, the more
stringent the procedural considerations must be. This is to avoid coercion or
exploitation, and to ensure that the disclosure of information has taken into account
the patient's capacity for understanding and evaluating the situation.

Equally, certain radical medical procedures call for stringent requirements to be met
on personal and procedural competence. Such preconditions may be specifically
formulated, as in Norway's abortion, sterilization, and transplantation legislation.
These preconditions reflect a general principle of international health law, illustrated
by the 1973 US case of Kaimowitz vs Michigan Department of Mental Health. Here
the court found that even though a difficult psychiatric patient possessed the
competence required to consent to ordinary surgical procedures, and even to
"accepted neurosurgical procedures", his competence would not be sufficient for him
to consent to experimental neurosurgical procedures characterized as "dangerous,
intrusive, irreversible, and of uncertain benefit to the patient and the society".

The requirement for informed and voluntary consent is not always applicable to non-
invasive medical procedures. In most cases, staff should be able to decide on the

background image

necessary procedures without having the patient's express consent, after having
provided information to the patient. They must, however, respect a patient's rejection
of any specific examinations or treatments. When the treatment offered is invasive,
the doctor will have the responsibility for providing the patient with all necessary
information--about the risks and the alternative treatments and their probable
consequences.

' .

&

&

+.,

&

&

J

+6,

)

&

&

'

+!,

(

)

)

+7,

(

)

)

+ ,

(

)

0

) )

3

'

#

)

!'!

)

)

'

8'4

&

& >

+., #

3

'

+6, #

2

&

&

3

'

+!, #

&

K

J

'

+7, #

(

)

)

)

0 &

'

background image

+ , #

'

#

)

'#

&

&

'

9'

)

&

+.,

&

&

&

+6,

&

&

&

'

+!,

&

&

&

( '

+7,

3

)

&

( '

+ ,

3

)

&

( '

#

)

7'.

&

&

)

3

'

<'4

&

)

&

)

1

)

&

>

+., .)

&

1

3

&

&

3

'

+6, .)

&

1

3

&

)

'

background image

+!, .)

&

1

3

&

&

)

0

'

+7, .)

&

1

3

&

&

)

0

'

+ , E &

1

3

&

)

'

#

)

.' .

&

&

)

'




background image

Resuscitation after severe hemorrhage is a very delicate process that involves many
biological procedures that need to be carried out within a very limited time window. To
successfully revive a victim of severe exsanguination, one must work quickly within the
"golden hour" time limit. In this time, there are various traumas that must be avoided;
cardiac arrest, hypotension, hemorrhagic shock, and hypothermia to name a few.

The resuscitation process involves the restoration of normotension (normative blood
pressure) while avoiding cardiac arrest and hemorrhagic shock. Even when normotension
has been restored, hemorrhagic shock, which can cause organ failure, can be a concern.
There are three types of hemorrhagic shock: compensated hemorrhagic shock;
uncompensated hemorrhagic shock, which is reversible; and irreversible hemorrhagic
shock. The most common method of resuscitation makes use of lactated Ringer's solution
to compensate for lost blood volume by causing the cells to swell, which in turn restores
normotension. Though this works fairly well, it is not the optimal treatment.

Half of the deaths that occur annually are due to acute illness or injury, and are associated
with circulatory failure or shock. Some of these deaths could be avoided by proper
monitoring. Present technology involves monitoring early in the temporal course of an
acute illness to observe the cardiac index, oxygen delivery and oxygen consumption. In
future, a possibility for a very effective non-invasive monitoring device, would be one
which could provide the following cardiac output readings: pulse oximetry for estimating
arterial hemoglobin oxygen saturations, a reflection of pulmonary function;
transcutaneous
oxygen and CO2 tensions, reflections of tissue perfusion; and noninvasive blood pressure
readings. Non-invasive systems are far more effective than invasive systems because they
provide a constant display of the data and can be used at any location, whether in the
hospital or in the field. In future, if systems like these are perfected, shock may be easily
intercepted and avoided, thus resulting in a significant number of saved lives.

Future studies on resuscitation should be centered around shock, prevention of cardiac
arrest as well as on increasing oxygen transport and increasing blood volume. The validity
of blood pressure as a measure of organ viability and optimum possibility of resuscitation
should be investigated as well. That said, resuscitation research should center mainly on

background image

developing procedures that can do all of these quickly and efficiently so that resuscitation
will save lives everywhere - from hospital to battlefield.

'

&

+.,

) &

&&

&

&

&

'

+6,

&

&

+!,

&&

&

+7,

&

&)

&

+ ,

&

#

)

.'#

& )

)

&&

)

'

8'#

&

)

3

& >

+.,

+6,

+!,

)

+7,

&

0

+ ,

- &

#

)

7';

&

&

&

0

&

&

&

)

'

9'.

&

&

&

)

&

& >

+., #

&

)

!

8

&

&

&

(

( &

!

8

'

background image

+6, #

&

)

( &

&

&

&

(

( &

!

8

'

+!, #

&

)

( &

&

&

&

(

( &

)

(

'

+7, #

&

)

( &

&

&

&

(

( &

!

8

'

+ , #

&

&

( &

&

&

(

( &

!

8

'

#

)

6'#

&

&

&

)

)

'

<'

)

&

K

&

J

+.,

+6,

&

+!,

+7,

&

0

'

+ ,

'

#

)

!'.

&

&

&

&

0

K

&

J

'

'

background image

1

&

)

.

1

&

.

*

&

)

&

;

()

'#

&

3

)

)

.

'E0

$

.

)

)

&

&

&

'

&

(

&

&

)

0

) '6

&

)

&

&

(

'.

&

&&

&

& )

'

I 0/

2

&.

$

'/

&&

)5

&

&

.

&&

.

&

)

'

#

)

&

&

1

4

&

'

&

.

&

&

&

&

'/

&

.

) 0

/

6

3

)

&

)

'

0 &

)

&

.

3

0

' .

'C

.

)

&

/

$

& '

' #

I 0/

$

0

+.,

)

&

+6, 3

(

+!,

(

)

background image

+7,

)

+ ,

'

#

)

7'

&

&

/

$

&

.

$

'

8'.

&

.

1

&

)

+.,

)

) ;

1

&

)

)

+6,

)

1

&

+!,

)

)

1

&

&

+7, 1

&

)

)

&

+ , 1

&

)

) ;

)

)

#

)

!'.

&

& 1

&

&

&

(

'

9'4

&

1 #

)5

&

.

>

+., - &
+6,
+!,

&

+7,

&

+ , )

0 &

#

)

'

&

&

. 6 !

7

)

&

.

'

<'#

$

&

&

+.,
+6,

background image

+!,

0

+7,
+ ,

#

)

7' #

5

&

&

'

background image

.

&

& &

&

&

)

&

'

&

&

&

)

&

&

'

)

'

L

)

)

0

&

( '

L

)

& 2

)

# 5

'.)

)

&

'4

)

&

(

$

'

0

'#

&

'#

&

'.# 5

+

/

0

,

&

0 0

&

&

'

%

&)

2 &

&

& '

)

)

& )

) '.

)

)

&

'. & )

)

&

)

&

'.

'#

*

&

)

'

6

&

*

&

&

'%

&

)

)

) 0

'


!

)

)

)

&

'

)

&

&

&

&

';

&

0

)

&

'%

)

&

&

)

0

0

'

'#

&

&

>

background image

.'!

0

&

'

6'

!

&

'

!'

!

0

&

'

7'

%

&

)

)

) 0

'

'

%

&

)

)

) 0

'

#

)

.'#

&

)

)

)

&

'

8'.

&

&

.'

'

6'

)

)

*

&

'

!'

&

0 )

&

'

7'

&

& )

)

'

'

*

&

0 )

&

'

#

)

!'!

.

6

)

'!

7

)

& )

)

)

'!

)

*

&

'

9'.

&

&

)

&

>

'

#

& )

&

'

#

&

0

'

#

&

&

.'
6'

!'

7'

'

#

)

7'L

)

&

)

& )

&

'

background image

<'.

&

&

&

.'# 5
6'

)

)

!'
7'

')

'

#

)

7'.

&

&

&

&

'

background image

Until now, efforts at preventing the proliferation of bioweapons have relied on

limiting access to the pathogens themselves. However, if terrorists can produce a

bioweapon from genome information alone, the same logic suggests that access to

genetic information should also be restricted. Scientists have begun to discuss the

restriction of the export of critical pieces of DNA. This measure could be extended to

the three big genome databases.

Scientists have long been aware of the potential dangers. The creation of polio, for

instance, was forecast months ago in an essay in Nature Immunology pointing out that

it would be simple to build an artificial polio virus. Nevertheless, most scientists still

oppose any attempts to restrict access to information. Earlier this year, the US

Department of Defense dropped proposals for checking any research it funds for

"sensitive" information before it is published, after scientists protested that this would

impede research needed to defend against bioweapons.

This was the case with anthrax. Unlike some other potential bioweapons, anthrax still

exists in nature, infecting animals and sometimes humans, and samples are held in

many labs worldwide. There is no reason for a terrorist to try to recreate it. The same

is not true of Ebola, smallpox or the 1918 flu virus. Even so, when the American

Society for Microbiology considered whether it should publish the smallpox genome,

it reasoned that the benefits in terms of understanding the virus and designing drugs

outweighed the risks.

Not all scientists share such views. Raymond Zilinskas of the Monterey Institute of

International Studies in California thinks some limits should be placed on the

publication of information on organisms such as smallpox. Zilinskas contends that

most scientists feel that basic research should not be restricted in this fashion. But

where does one cross the line? He and others have proposed that professional

societies and editorial boards at scientific journals should exert more control.

Even if they do not, the decision could be taken out of their hands. In addition to

international measures, individual governments are also cracking down. The USA

Patriot Act passed this year allows the federal government to stop some foreign

nationals working in the US from getting access to certain pathogens and toxins. The

US could extend this to cover access to genetic sequences as well.

1. The passage is primarily concerned with

(A) detailing the evidence that supports the claim that genetic information should be

restricted

(B) outlining the factors that have contributed to the restriction of genetic information

(C) evaluating whether sensitive genetic information should be restricted in light of

the danger of bioterrorism.

(D) summarizing the differences between genetic research in free countries and such

research in countries that promote terrorism.

background image

(E) formulating a hypothesis about the proliferation of bioweapons.

The best answer is C. The passage does not reach any conclusions, it simply

evaluates the question of limiting access to sensitive genetic information.

2. Terrorists would be unlikely to try to steal genetic information about anthrax

because

(A) it is difficult to control and could therefore be a threat to the terrorists

themselves.

(B) they do not have the necessary know-how to utilize genetic information to build

anthrax powder

(C) actual anthrax is not only available in nature, but also in laboratories.

(D) the American Society for Microbiology has limited the dissemination of genetic

information on anthrax.

(E) the American Society for Microbiology has published genetic information on

anthrax.

The best answer is C. Since anthrax itself is attainable, it is unlikely that bioterrorists

would try to synthesize it using genetic information.

3. All of the following statements are supported by the passage EXCEPT:

(A) the smallpox genome has been published by the American Society for

Microbiology

(B) federal government has stopped some foreign nationals from accessing to

certain toxins

(C) Raymond Zilinskas of the Monterey Institute believes the anthrax genome should

not be published

(D) Most scientists oppose any attempts to restrict access to information

(E) it would not be difficult for a bioterrorist to build an artificial polio virus

background image

The best answer is C. According to the passage, Raymond Zilinskas believes some

limits should be placed on the publication of information, but one cannot determine

from the passage if that would include anthrax information.

4. It can be inferred from the passage that which of the following is true?

(A) The USA Patriot Act will eventually stop all foreign workers from access to

genomes.

(B) It is not known whether or not bioterrorists have to skill to produce an actual

bioweapon from genome information alone.

(C) The Ebola virus affects animals.

(D) According to Nature Immunology magazine, some terrorists have built an

artificial polio virus

(E) The three big genome databases have restricted of the export of critical pieces of

DNA.

The best answer is B. In the first paragraph of the passage it is stated “if terrorists

can produce a bioweapon from genome information alone”. One can infer from this

that it is not known whether the terrorists can produce one or not.


Wyszukiwarka

Podobne podstrony:
reading comprehension
READING COMPREHENSION
33 Teaching reading comprehension
reading comprehension UZSTU65BT6P3Z7ZNNSS5CQTZ732IDDEISI54UGQ
LearningExpress Reading Comprehension Success in 20 Minutes a Day 3rd
Learningexpress Reading Comprehension Success 3rd Edition
LearningExpress 501 Reading Comprehension Questions 3rd
Longman Stories for Reading Comprehension 2
Hayati, Shoohstari, shakeri Using humorous texts in improving reading comprehension of EFL learnesr
reading comprehension jack london worksheet
reading comprehension dr dolittle worksheet
thanksgiving reading comprehension
US Army course Reading Comprehension, Study Methods, and Time Management Skills PD2201
islcollective worksheets preintermediate a2 elementary school reading halloween reading compreh en 1
reading comprehension dr dolittle answersheet
reading comprehension whale throat worksheet worksheet

więcej podobnych podstron